APSC Prelims 2023 General Studies GS & CSAT Paper 1 & Paper 2 Answer Key Download PDF

APSC Prelims 2023 General Studies (GS) & CSAT Paper 1 & Paper 2 Answer Key Download PDF

APSC prelims questions

 

APSC Prelim 2023 GS-I Paper Answer  Key Download PDF

APSC Prelim 2023 GS-I Paper – Download PDF

APSC Prelim 2023 CSAT Paper 2 Answer  Key Download PDF

APSC Prelim 2023 CSAT GS-II Paper – Download PDF

Go to APSC Prelim Previous Years Questions

 

 

Assam Current Affairs – January 04-06, 2024

Assam & NE Current Affairs & GK – January 04-06, 2024

( Covers all important Current Affairs & GK topics for the January 04-06, 2024)

Go To Assam Current Affairs                      Go To Assam Current Affairs Quiz

Current Affairs Assamexam

January 04-06, 2024 

North Eastern Development Finance Corporation Ltd (NEDFi) sanctions over ₹750 crores for 5,300 Northeast Projects

North Eastern Development Finance Corporation Ltd (NEDFi) has sanctioned Rs 766.47 crore and disbursed Rs 547.11 crore for 5338 projects in the NorthEast region, in diverse sectors – education & training food processing, handlooms & handicrafts, healthcare, hotels & tourism, microfinance etc.

Important Initiatives of NEDFI

  • NEDFi in collaboration with the Ministry of Development of North Eastern Region (MDoNER) in setting up ₹100 crore North East Venture Fund to promote regional startups.
  • It also partners with banks, microcredit organisations and NGOs for last-mile delivery to remote areas.
  • North Eastern Development Finance Corporation Ltd (NEDFi) plays an important role in facilitating setting up of new industrial and service sector projects in the NE Region.
  • ₹547 crore disbursed at concessional interest rates to micro and small businesses 
  • ₹41 crore credit extended via microfinance institutions ₹53 crore direct micro lending to benefit over 9 lakh individuals.
  • ₹98 crore venture capital commitments to 67 startups.

About North Eastern Development Finance Corporation Ltd (NEDFi) 

  • It is a public limited company registered under the Companies Act 1956 on August 9, 1995.
  • It provides financial assistance to MSMEs in the NE region for setting up industrial, infrastructure and agriculture-related projects. 
  • It also provides micro financing through MFIs/NGOS. It also provides consultancy and advisory services to state governments, private sectors and other agencies.

Three New Frogs Discovered in Arunachal Pradesh

The Scientists have reported three new frog species belonging to new genera from Arunachal Pradesh’s Namdapha-Kamlang protected area network along the Myanmar border.

About the discovery 

  • Gracixalus patkaiensis – A translucent green tree frog with jelly-like appearance and insect-like call
  • Alcalus fontinalis – A tiny brown frog inhabiting fast-flowing hill streams with unique trickling vocalization 
  • Nidirana noadihing – A marsh-dwelling “music frog” named after the Noa-Dihing river

About Namdapha Tiger Reserve

  • It is located in the Changlang district of Arunachal Pradesh, near the international border with Myanmar.
  • It was declared a Wildlife Sanctuary under the Assam Forest Regulation 1891 on 2nd Oct.’1972. 
  • In the 1983 year it was declared as a tiger reserve under Project Tiger Scheme of the Govt. of India on 15th March 1983.

India, Nepal hold Seventh Joint Commission Meeting

The Seventh Meeting of the Nepal-India Joint Commission was held in Kathmandu. External Affairs Minister S Jaishankar and his Nepal counterpart NP Saud recently attended the 7th meeting of the Nepal-India Joint Commission.

 

Key Points

  • Nepal and India signed a long-term agreement for the export of 10,000 MW of power to India for the next 10 years. 
  • Inauguration of Cross-Border Inauguration of 3 cross-border transmission lines which was completed with the help of India (132 kV Raxaul-Parwanipur, 132 kV Kushaha-Kataiya and New Nautanwa-Mainahiya lines). 
  • An agreement for the launch of Nepali satellites was formalized between the Nepal Academy of Science and Technology and NewSpace India Limited under the Indian Space Research Organisation.
  • Inaugurated 3 132-kV cross-border transmission strains, which include the second circuits of the Raxaul-Parwanipur line and the Kataiya-Kusaha line, and the New Nautanwa-Mahiya line.

About India-Nepal Joint Commission: 

It was established in 1987 as a platform for the Foreign Ministers of both countries to review all aspects of the bilateral partnership.

Raghuram Iyer appointed as CEO of Indian Olympic Association 

Raghuram Iyer has been appointed as the Chief Executive Officer (CEO) of the Indian Olympic Association (IOA).

His appointment follows a meticulous selection process conducted by the nomination committee of the IOA.

The appointment comes after a significant delay and repeated reminders from the International Olympic Committee (IOC). 

Raghuram Iyer’s extensive experience in sports management and administration, particularly his previous roles as CEO of Indian Premier League teams Rajasthan Royals and Rising Pune Supergiants, has been recognized as making him an ideal candidate for this position.


XPoSat (X-ray Polarimeter Satellite) mission

The Indian Space Research Organisation successfully launched the PSLV-C58 XPoSat (X-ray Polarimeter Satellite) mission.

About XPoSat (X-ray Polarimeter Satellite)

  • XPoSat is designed to study X-ray polarization in the medium X-ray band, offering insights into celestial sources’ radiation mechanisms and geometry. 
  • This study is crucial for understanding the physics behind these celestial bodies.
  • The satellite carries two main payloads, POLIX (Polarimeter Instrument in X-rays) and XSPECT (X-ray Spectroscopy and Timing).
  • It is India’s first dedicated polarimetry mission to study various dynamics of bright astronomical X-ray sources.
  • It is designated for observation from low earth orbit.
  • After this launch, India became the second nation after the US to send an observatory to study astronomical sources, such as black holes, and neutron stars, among others.

Wetland City Accreditation

The Ministry of Environment, Forest and Climate Change has submitted three nominations from India for Wetland City Accreditation (WCA).

The nominated cities include Indore (Madhya Pradesh), Bhopal (Madhya Pradesh), and Udaipur (Rajasthan).

 These are the first three Indian cities for which nominations have been submitted for WCA based on the proposals received from respective State Wetlands Authorities in collaboration with the Municipal Corporations.

About Wetland City Accreditation:

  • The Ramsar Convention during COP12 held in the year 2015 approved a voluntary Wetland City Accreditation system.
  • The scheme aims to promote the conservation and wise use of urban and peri-urban wetlands, as well as sustainable socio-economic benefits for local populations.
  • It recognizes cities which have taken exceptional steps to safeguard their urban wetlands and also recognizes the importance of wetlands in urban and peri-urban environments and to take appropriate measures to conserve and protect these wetlands.
  • This voluntary scheme provides an opportunity for cities that value their natural or human-made wetlands to gain international recognition and positive publicity for their efforts.

About the cities nominated for WCA

  1. Indore: Founded by Holkars. Sirpur Lake, a Ramsar Site in the city has been recognised as an important site for water bird congregation and is being developed as a Bird Sanctuary. 

A strong network of wetland mitras is engaged in bird conservation and sensitising the local community to protect Sarus Crane. 

  1. Bhopal: Bhoj Wetland, Ramsar Site is the city’s lifeline, equipped with the world-class wetlands interpretation centre, Jal Tarang. Additionally, the Bhopal Municipal Corporation has a dedicated Lake Conservation Cell. 
  2. Udaipur: The city is surrounded by five major wetlands, namely, Pichola, Fateh Sagar, Rang Sagar, Swaroop Sagar, and Doodh Talai. These wetlands are an integral part of the city’s culture and identity, help maintain the city’s microclimate, and provide a buffer from extreme events.

Union Cabinet approves overarching scheme “PRITHvi VIgyan (PRITHVI)”

The Union Cabinet recently approved the comprehensive scheme “PRITHvi VIgyan (PRITHVI)” by the Ministry of Earth Sciences. This overarching initiative, with an allocation of Rs 4,797 crore for the period 2021-26.

About PRITHvi VIgyan (PRITHVI) Scheme

Objectives:

The major Objectives of the overarching Prithvi Scheme are

  • Augmentation and sustainance of long-term observations of the atmosphere, ocean, geosphere, cryosphere and solid earth to record the vital signs of the Earth System and change 
  • Development of modelling systems for understanding and predicting weather, ocean and climate hazards and understanding the science of climate change 
  • Exploration polar and high seas regions of the Earth towards discovery of new phenomena and resources; 
  • Development of technology for exploration and sustainable harnessing of oceanic resources for societal applications ·
  • Translation of knowledge and insights from Earth systems science into services for societal, environmental and economic benefit.

The Prithvi scheme integrates five existing sub-schemes: 

  1. Atmosphere and Climate Research-Modelling Observing Systems and Services (ACROSS) 
  2. Ocean Services, Modelling Application, Resources and Technology (O-SMART) 
  3. Polar Science and Cryosphere Research (PACER) 
  4. Seismology and Geosciences (SAGE) 
  5. Research, Education, Training, and Outreach (REACHOUT).

Additionally, the scheme focuses on providing advanced warning services for natural disasters like cyclones, floods, heatwaves, and earthquakes, facilitating prompt and effective disaster management. 

The scheme ensures precise weather forecasts for both land and oceans, enhancing safety and minimizing property damages in adverse weather conditions.


‘Clean and Green Village’ Programme  for Solar Power Expansion

The Ministry of Panchayati Raj (MoPR) is prioritizing the ‘Clean and Green Village’ theme as part of its Gram Panchayat Development Plan (GPDP). 

About ‘Clean and Green Village’ Programme

  • Clean and Green Village’ is the fifth theme under the GPDP.
  • The programme aims to install solar power capacities on a commercial basis. 
  • It aims to develop sustainable business models for solar initiatives at the grassroots level. 
  • The programme focuses on access to safe water, sanitation, and low pollution environments for healthier lives. 
  • It emphasizes sustainable natural resource management for better livelihoods, food security, and environmental protection.
  • It aims to achieve UN Sustainable Development Goals (SDGs), particularly SDG7 (affordable and clean energy) and SDG13 (climate action). 

Areas of Action: Includes 10 specific areas like Open Defecation Free (ODF) Village, Clean and Green School, promotion of Organic Farming, and more.

Initiatives and Schemes for Clean and Green Village: 

  • Organic and natural farming initiatives like Paramparagat Krishi Vikas Yojana. 
  • Reducing chemical fertilizers through schemes like PM-PRANAM. Waste to wealth initiatives like GOBARdhan and SATAT.

Eaglenest Wildlife Sanctuary

Researchers at the Indian Institute of Science (IISc.) in Bengaluru have observed a significant shift in the habitat of numerous bird species in the Eaglenest Wildlife Sanctuary due to the escalating temperatures in the region.

About Eaglenest Wildlife Sanctuary

  • It is a protected area of India in the Himalayan foothills of West Kameng District, Arunachal Pradesh. 
  • It conjoins Sessa Orchid Sanctuary to the northeast and Pakhui Tiger Reserve across the Kameng river to the east. 
  • It is situated in the biodiversity hotspot of Eastern Himalayas and home to over 500 bird species.
  • It is also a part of the Kameng Elephant Reserve. 
  • It derives its name from the Red Eagle Division of the Indian army The temperate cloud forest is intermixed with dense bamboo patches and broadleaved evergreen forest across a wide altitudinal range, with conifers and rhododendrons at the higher elevations.

Assam Govt and Indian Govt sign peace Pact with ULFA (All Provisions) – Assam/Northeast Polity Notes

Assam Govt and Indian Govt sign peace Pact with ULFA (All Provisions) - Assam/Northeast Polity Notes
Assam Polity - Assamexam

Go To Assam Polity Notes & Study Materials                                                                Go To Assam Polity Quiz MCQ

Recently, the Union Ministry of Home Affairs (MHA), the Assam government and the pro-talks faction of the United Liberation Front of Asom (ULFA) signed a memorandum of settlement (MoU). The agreement was signed with the pro-talks faction of ULFA under Arabinda Rajkhowa while the other faction led by Paresh Baruah (known as ULFA-I) has not joined the peace process.

Major provisions in the ULFA Peace Accord

  1. Development Measures- The peace agreement encompasses a comprehensive package for the all-round development of Assam. It includes provisions for various developmental projects and a commitment to resolving long-standing issues. The government has pledged investment of ₹1.5 lakh crore, staggered through the years.
  2. Political Inclusion- The agreement addresses the issue of political insecurity among indigenous people, reserving 97 out of 126 seats for them in the recent delimitation exercise. It ensures the continuation of this principle in future delimitation processes, aiming to address political insecurities.
  3. Non-Violence- ULFA has agreed to renounce violence, disarm, disband the armed organization, vacate their occupied camps, and participate in the peaceful democratic process established by law. This commitment aims to ensure the integrity of the country and marks a shift towards non-violent engagement.
  4. A time-bound program will be made by the Ministry of Home Affairs, Government of India, to fulfil the demands of ULFA and a committee will also be formed for its monitoring.
  5. Govt. of India and the Govt. of Assam remain committed to protect the interests of the indigenous communities, ULFA agreed to shun the path of violence, join the peaceful democratic process as established by the law of the land, and uphold the integrity of the country. 

 

Political Demands

  1. The Govt. of India agrees to maintain the territorial integrity of the State of Assam. However, the Govt. of India and the Govt. of Assam are committed to amicably resolve boundary disputes of Assam with neighbouring States.
  2. The Govt. of India will consider recommending to the Election Commission of India to follow, as far as possible, the broad guidelines and methodology, which was adopted for the delimitation exercise held in Assam in 2023, in future delimitations as well. Some of these parameters used in the exercise are rate of growth of population, density of population, principles of historicity of areas and communities, geographical continuity, physical features, facilities of communication and public convenience. 
  3. Effective measures will be taken to prevent enrolment of illegal migrants in the voter lists. All precautions will be taken to ensure thorough scrutiny of new voters at the time of revision of electoral rolls. The verification exercise should be carried out under the direct supervision of the District Electorar Registration Officer.
  4. ST Status for Communities: The Constitution (Scheduled Tribes) Order (Amendment) Bill, 2019 has been introduced by the Govt. of India in the Rajya Sabha to include certain communities in the list of the STs in relation to the State of Assam. The Govt. of Assam has constituted a Committee of Group of Ministers (CGoM) to examine and suggest on the different aspects of reservation of six communities in the State. The Govt. of India and the Govt. of Assam will make efforts to expedite the process after signing of the MoS.
  5. Reservation in Education/Employment: The Govt. of Assam agrees to consider initiating measures to divide OBCs/MOBCs into separate categories and specify quantum of reservation for each category for the purpose of reservation in educational institutions and employment under the State Govt. The State Govt. has already constituted the Assam Backward Classes Commission in April 2022 under the Assam Backward Classes Commission Act, 1993 towards this objective.
  6. NRC/Illegal Immigration: Foreigners detected after preparation of the corrected and error-free NRC shall be dealt with as per the law of the land. Effective measures, including use of latest state of the art technologies, will continue to be taken jointly by the Govt. of India and the Govt. of Assam to secure international borders to prevent fresh illegal influx into the country.
  7. Land/Forest: For efficient management of land resources, the Govt. of Assam will take steps to digitalise all land holdings, preferably within 3 years of signing of the MoS. The Govt. of Assam will take necessary steps to prevent shrinkage/decrease of prime agricultural land. No agricultural land will ordinarily be allotted or settled for any other purpose except when the State Govt. is of the view that the intended purpose is in public interest such as establishment of industry for employment generation, construction of public institution/office, hospital, dispensary, etc.
  8. Tea Gardens: The Govt. of Assam will consider constituting a Committee under the Chairmanship of a Cabinet Minister to examine the issue of sick tea gardens and recommend steps as to how to revive them. The Committee will also study various issues relating to welfare of the tea garden workers and recommend appropriate measures to address their grievances, in consultation with all stakeholders including the plantation owners. Persons belonging to Tea and Ex-Tea garden community will be given preference in allotment of land acquired from Tea Estate under the Assam Fixation of Ceiling on Land Holdings Act, 1956, as amended from time to time. 
    Efforts will be made by the Govt. of India to re-locate the Office of Tea Board of India from Kolkata to a suitable location in Assam.
  9. The Govt. of India may consider facilitating acquisition of land at Delhi and provide financial assistance for constructionn of a Community Centre, which will inculcate the spirit of Srimanta Sankardev and Lachit Borpukhan, at the national level.
  10. The Govt. of India may consider setting up a Visual Art University in Assam to promote film production, performing arts, animation etc.
  11. The Govt. of India may consider establishing a Regional Centre of Lalit Kala Academy for the North East in Assam.
  12. Govt. of Assam will set up an Archaeology Research Centre at Tezpur.
  13. An Ethnic Museum will be set up by the Govt. of Assam in Dhemaji, which was the second capital of Ahom kingdom (Dhemaji Habung).
  14. A heritage centre will be set up by the Govt. of Assam at Jamugurihat (Sonitpur district) for preserving the “Bhaona Culture”.
  15. Cultural Academy and Research Centres will be set up by the Govt. of Assam in erstwhile Lakhimpur, Darrang, Goalpara (to be named after Pratima Pandey Baruah), Kamrup, Sivasagar, Cachar and Nagaon districts, which are known for their rich cultural tradition since pre-Independence days.
  16. The Govt. of India will take up a number of developmental projects in Assam in the next few years covering core infrastructure sectors such as Road Transport & Highways, Railways, Flood and Soil Erosion, Petroleum & Natural Gas, MSME, Textiles, MEITY, etc. A list of identified projects is at Annexure — I. 10.2 The Govt. of Assam will also take up a number of developmental projects in the State in the next few years covering various sectors such as Energy, Petroleum & Natural Gas, Roads & Bridges, Floods and Soil Erosion, Wetland Development, Education, Cottage Industry and Delivery of services to improve ease of living. A list of identified projects is at Annexure — II.
  17. Special Development Package: A Special Development Package of Rs. 5000 Crores will be provided over a period of five years with matching contributiorf by the Govt. of India and the Govt. of Assam for various developmental initiatives in the State.

 

ROAD TRANSPORT AND HIGHWAYS 

A. ROAD

1. Highway from Diphu to West Karbi Anglong via Hamren 

2. 4-lane Gauripur Bypass from Dumardohan to Baladmara via Khudimari, Fulkumari, Charaldanga, Sajuarkuti, Geramari 

3. Two lane road from Majuli to Jorhat on NH-715K 

4. Stage strengthening of NH-127C from Samthaibari to Dadgiri 

5. New Bridge over River Gai on NH-15 

6. River Training Work for New 2-Lane Major Bridge between Majuli and Jorhat

7. Construction & up-gradation of NH-127C from Samthaibari to Galengphu (Indo-Bhutan Border) 

8. Widening of NH-329

 9. Tamenglong – Mahur Road from Jiri River to Hangrum 

10. 4-Laning of Bilasipara-Tulungia and Silchar-Jiribam 

11. 4-Laning of Silchar ISBT of NH-37 and NH-6 

12. 4-Laning of NH-37/NH-8 near Sutarkandi (Spur Connectivity to Indo/Bangladesh Border) and proposed Nilam bazar/ Cheragi Bypass on Silchar-Churaibari 

13. 4-Laning of NH-8 (proposed Nilambazar/Cheragi Bypass near Chandkhira Village and near Churaibari Village (Assam/Tripura Border) on Silchar-Churaibari 

14. 4-Laning of Malidor-Ratachera-Kalain-Badarpur 

15. 4-Laning of Silchar-Jiribam 

16. Construction of new highway from NH-37 near Pailapool to NH-137 connecting Jaipur, Harinagar, Kumarcherra, Zinam, Hagrum (Mahur-Tamenglomg Road) 

17. (a) 4-Lane Approach road from Numaligarh side to Brahmaputra Tunnel on South Bank

 (b) 4-Laning of Brahmaputra Tunnel Connectivity across River Brahmaputra between Gohpur (on NH-15) and Numaligarh (on NH71) 

(c) 4-Laning of Approach road from End of Brahmaputra River Tunnel to Gohpur side on North Bank 

18. Guwahati Ring Road including Brahmaputra Bridge 

19. Implementation of wild life friendly measures on Kaziranga National Park stretch of NH-715 Baihata to Sonapur section) and (Basistha to Jorabat section) on NH 37 

B. Railways 

20. World Class Station Building at Kamakhya Railway Station 

21. New Railway line Salona-Khumtai 

22. New Railway line Tezpur-Silghat 

23. Saraighat Bridge doubling 

24. Doubling of Kamakhya — New Guwahati — 3rd Line 

25. Doubling of Lumding – Tinsukia Jn – Dibrugarh railway line 

26. Multi-tracking of Azara- Tetelia 

27. Multitracking from Khandikar- Kendukona (Rangia By-pass) 

28. Chandranathpur — Agartala 

29. Agthori – Dekargaon (New line)

 30. New lines : Ledo – Jairampur- Kharsang; Gogamukh—Jorhat; Jowai-Khlieriat— Chandranathpur; Kokrajhar-Gelephu 

C. FLOOD AND SOIL EROSION 

31. Anti-erosion measures at different reaches of river Kushiyara 

32. Raising and strengthening of Dyke from Bahari to Baghbar

33. Anti-erosion measures at different reaches of river Garuphella

 34. Anti-erosion measures at different reaches of river Longa 

35. Protection of Majuli island from flood and erosion of river Brahmaputra 

36. Integrated Water Resources Management of Buridehing Basin 

37. Integrated flood and erosion management of Manas and Beki rivers in Baksa and Barpeta districts 

38. Integrated flood and river bank erosion manaaement works at Rohmoria 

39. Protection of Flood embankment along Kollong river from Raha to Jagi

 40. Construction of Dyke along right bank of river Kollong from Dhankhunda to Gobardhan hill and sluice gate at Dhepuijan, Ghoramarajaan and Kajolijan 

41. Protection of Kolbari and Lality area from the erosion of river Brahmaputra (Construction of three nos of spurs and a check dam) 

42. Protection of various villages from the erosion of river Aie 

43. Rejuvenation, bioremediation and conservation of Silsako Beel at Guwahati 

D. PETROLEUM & NATURAL GAS 

44. Pipeline from Duliajan to Majuli 

45. Laying of pipeline from Narayanpur, Assam to Banderdewa & Naharlagun in Arunachal Pradesh 

46. Pipeline from Guwahati, Assam to Sandrup Jonkhar, Bhutan

 47. Pipelines from Silchar, Assam to Imphal, Manipur via Jiribam 

48. Pipeline from Baihata, Assam to Panikhaiti, Assam and from Sonapur, Assam to Tura & Mairang, Meghalaya. 

49. Capacity augmentation of Bongaigaon LPG Plant 

50. Laying of Guwahati-Silchar-Imphal Product Pipeline (GSIPL)

51. Bongaigaon Refinery Expansion 

52. Geleki Redevelopment Project 

53. Creation Of Captive Power Plant (CPP) at Geleki 

54. Creation of Surface Facilities and Redevelopment of Geleki Field 

55. Pipeline Network Project, Rudrasagar

56. Creation of Captive Power Plant (CPP) at Rudrasagar 

57. Pilot implementation of Chemical Enhanced Oil Recovery (CEOR) in Rudrasagar, Lakwa, Lakhmani 

E. MSME

58. Development of New Industrial Estate at Chenga, Bahari, District Barpeta

59. Setting up of a new industrial estate at 2, Dhemajibari, Biswanath Cherialx

60. Up-gradation of the Integrated Infrastructure Development Centre IIDC, Silapathar, District Dhemaji

61. Development of New Industrial Estate at Maizgram, District Karimganj

62. Up-gradation of the Integrated Infrastructure Development Centre, Titabor, Jorhat

63. Development of New Industrial Estate at Dighalichapari, District-Sonitpur

64. Infrastructure Development Facilities & Up-gradation in the existing Complex, Manja, Karbi Anglong, Assam.

F. TEXTILES

65. Eri and Muga Silk Development Project for creation of livelihood of forest communities

66. Integrated project for Eri Silk Industry

67. Project for empowering women from forest dependant communities in Soil to Silk value chain (Ava Creation – SPV)

G. MEITY

68. Research Centre for Nano-technology. 

Annexure-!!

List of Developmental Projects to be taken up by Govt. of Assam 

A. ENERGY

1. Development of 3000 MW Solar power projects in Assam

2. Increased Hydro-Power generation by expansion of KarbiLangpi Project with additional 50 MW

3. Develop Assam as a region hub by establishing 250 MW Battery Energy Storage.

4. Increased power supply in the State by exploiting green sources by developing 212 MW Solar Parks in the State

5. Development of anew 25 MW Namrup Solar Power Project 

6. Development of 20 MW Sonbeel Solar Power Project at Karimganj

B. PETROLEUM AND NATURAL GAS

 7. Increased oil and gas business in Assam in the midstream and downstream by expanding the capacity and diversification of NRL into 9 MMPTA

C. ROADS AND BRIDGES

8. Improved rural connectivity by construction of new and strengthening of Resilient Rural Bridges in the State 

9. Improvement of communication by taking up of Assam Bridge project connecting Palashbari

10. Special focus and investment for development of Roads in Hill Areas in Assam with suitable designs for Disaster Resilience

D. FLOODS AND SOIL EROSION

11. Protection of lives and livelihoods of people of Assam in flood affected areas by taking up Integrated River Basin Management Projects

12. Long term and sustainable development of Brahmaputra riverine areas by taking up projects like Climate Resilient Brahmaputra Integrated Flood and River Erosion Risk Management Programme

13. Climate Resilient Brahmaputra Integrated Flood and Riverbank Erosion Risk Management Project in: (a) Assam-Dibrugrh and Tinsukia Sub Project (Zone-A) (b) Assam-Morigaon Nagaon Tezpur Sub project (Zone-B) (c) Assam-Guwahati West P:G.P Sub project (Zone-C) (d) Assam-Barpeta Goalpara Dhubri sub project ( Zone D)

E. MISCELLANEOUS

14. Systemic transformation of Governance & Service Delivery in Assam to make it more pro-people and accountable by targeted investments

15. Focussed Investment to develop modern Urban Infrastructure and Urban Sector in Assam to promote Ease of Living

16. Develop and Implement time-bound program to cover all major towns of Assam with urban gas distribution to provide connections to all households in 5 years

F. WETLAND DEVELOPMENT

17. Eco-Conservation and Development of Assam Wetlands and Integrated Fisheries in a sustainable manner G. EDUCATION

18. Mission Mode and focused efforts and investments to support School education in Assam

H. COTTAGE INDUSTRY

19. Bell metal, Brass metal and charcoal depot based industries to be established in Sarthebari (Bajali District). State Govt. to look into the problems of easy availability of raw material to ensure survival of Bell based Assamese handicraft cottage industry. 

Annexure-III

Illustrative list of developmental projects to, be implemented underSpecial Development Packaqe,

A. EDUCATION

1. IIM at Guwahati subject to laid down govt. norms

2. School of Planning and Architecture subject to laid down govt. norms

3. Indian Institutes of Science Education and Research (IISER) subject to laid down govt. norms

4. A University in the name of Su-Ka-Pha in Charaideo district subject to aid down govt. norms

5. Education Park at Guwahati to impart interacting, demonstrative and physical education to youth on physical science, life and social sciences.

6. Training School for imparting hands on training in Oil and Gas operations

B. RAILWAYS

7. North Eastern Frontier Railways to be re-constituted to cover only the States of North East Region subject to laid down norms of Railway Board.

8. Railway equipment manufacturing plant at Bongaigaon

C. FLOOD AND SOIL EROSION

Flood and erosion problem will be treated as a national priority

D. INDUSTRY

10. Food processing Industrial park in Malinibari of Silchar District.

11. Special Economic Zone at Daranga Mela area of IndoBhutan border (Baksa District).

12. !ndustrial area for value added production from Banana in Goalpara district

13. Industrial area for value added production from Pineapple in Karbi Anglong district

14. A bamboo-based industry in Barak Valley

E. TOURISM

15. Development of Kaziranga as a high-end national level tourist attraction centre with various entertainment amenities, Botanical and Floral Garden, Orchid Centre, Eco Forest Museum and opportunities to showcase local culture, craft, cottage industry, etc.

16. Tea Museum at Dibrugarh

17. Development of tourist spots at .Rangghar, Talatalghar, Karengghar, Khaspur ruins, Brahmaputra waterfront at Guwahati, battlefield of Saraighat war and Bogeebil.

18. Development of water sports activities at Bogeebil (Dibrugarh District).

E. TOURISM

15. Development of Kaziranga as a high-end national level tourist attraction centre with various entertainment amenities, Botanical and Floral Garden, Orchid Centre, Eco Forest Museum and opportunities to showcase local culture, craft, cottage industry, etc.

16. Tea Museum at Dibrugarh

17. Development of tourist spots at . Rangghar, Talatalghar, Karengghar, Khaspur ruins, Brahmaputra waterfront at Guwahati, battlefield of Saraighat war and Bogeebil.

18. Development of water sports activities at Bogeebil (Dibrugarh District).

F. SPORTS

19. A multi-facility sports complex with Centre of Excellence in Volleyball at Nalbari

20. Centre of Excellence for Foot+oail in Dibrugarh and Baksa districts

21. Multi-purpose sports complex at Darrang and Charaideo districts

22. Football Academy at Kaiiabor (Nagaon District).

G. AGRICULTURE

23. An in ernationaliy accredited Organic Produce Certification

Centre to be set up in Assam Agriculture University.

24. A Regional Pesticide Testing laboratory to be set up at Guwahati.

25.  Muga Parks at Lakhimpur, Dhemaji, Sivasagar and South Ka,mrup with special incentives to Muga producers. Rural areas (Dhakuakhana, Dhemaji, Charaideo and Boko) to be declared as eco-sensitive zone for survival of Muga silk culture.

26. Setting up of a Germplasm Bank in Horticulture Collage at Nalbari (Lokhopur) for development of indigenous verities of flora and fauna and collection and preservation of indigenous/heirloom seeds.

27. Solar Cold Storage in Sadia

H. ACT EAST POLICY

28. An international cultural centre in Guwahati to promote people to people socio-cultural contact with neighbouring countries.

29. An International level amusement park to be set up near Mayang and Pabitora Wildlife Sanctuary area in collaboration with private sector.

1. SKILL DEVELOPMENT

30. Skill Development Centres will be set up to function in close coordination with industry and impart specialised training to the youth for enhancing their capabilities in areas that are in demand overseas such as hospitality, healthcare, construction, manufacturing etc.

J. BlO-FUEL SECTOR

31. A Committee will be constituted to study and recommend on promotion of investment and formulation of policy in consultation with Niti Aayog to make Assam a hub for green energy.

32. Green Banks will be set up to provide financial assistance for green energy projects.

K. WETLAND DEVELOPMENT

33. Development of eco-tourism and state of the art water sports hub along with fresh water fisheries at Pahu-Garh (Sivasager district), Kalang River (Nagaon district) and Deuri Silabandha bil (Kaliabar), From Bouta bil to Kapfa bil Wetland (hiafbariBarpeta district), Dheer Bill (Dhuburi district), MatapungMaguri bill (Tinsukia district) and Bagibill of Dibrugarh district.

L.  ROADS AND BRIDGES

34. Bridges at (i) Boko-Mukalmua (ii) Disangmukh-Matmora (Sivasagar district-Lakhimpur district) (iii) Lahorighat (Morigaon district) – Udalguri (Udalguri district), and (iv) Mora Deopani-Amarpur Telia Bari to be constructed.

35. Construction of road from Khatkhati to Namrup Dilighat bridge along Assam-Nagaland boundary.

36. Reopening of road communication on Goalpara – Dalu – Nalitabari axis (road along Indo-Bangladesh boarder which links different border trade points).

 

Peace Pact with ULFA: A Path Towards Peace

  • 2005: United Liberation Front of Assam (ULFA) formed an 11-member ‘People’s Consultative Group’ (PCG)and the committee mediated three rounds of talks.
  • 2008: Some ULFA commanders like Arabinda Rajkhowa strive for peace talks while Paresh Baruah was opposed and was expelled from Rajkhowa outfit leading to split in ULFA.
  • 2012: The pro-talks faction submitted a 12-point charter of demands (spanning constitutional, political, financial, and cultural concerns), which was finally responded to in 2023.
  • 2023: Peace Accord between Rajkhowa’s faction and the Centre was followed, culminating in the tripartite peace agreement.
  • One of the major key points is that 97 out of 126 Assembly seats would be reserved for indigenous people and the future delimitation exercise would follow this principle.

 

Significance of the Peace Pact

  • Progress and Development: ₹1.5 lakh crore investment had been pledged in the peace accord.
  • Political Will and Implementation: A time-bound program will be made by the Home Ministry to fulfill the demands of ULFA.
  • Ensuring Peace Assam: With an aim to achieve resolution and closure.
    Violent Groups Surrendered: As more than 9000 cadres have surrendered on record.
  • Triumph of Democratic Process: ULFA has also agreed to engage in the peaceful democratic process established by law and maintain the integrity of the country.

 

Way Forward

  • Fulfill Promise: Government needs to fulfill the promises made during the peace agreement, addressing the concerns and aspirations of the United Liberation Front of Assam (ULFA) and the affected communities.
  • Complete Peace Process: Ensure a comprehensive and thorough peace process.
  • Complete Integration and Assimilation: By involving rehabilitation programs, vocational training, and support for their social and economic integration.
  • Ensure Continued Monitoring: To ensure that all parties adhere to their commitments.
  • Neutralizing ULFA-1: Through collaboration with the Myanmar government. Diplomatic channels must be leveraged to counter any support provided to ULFA-1 by China.

The peace accord is a positive development for Northeast peace. However, challenges persist with the non-participating ULFA-I and cross-border issues. Sustained commitment and diplomatic efforts are essential for ensuring lasting stability in the region.

 

Go To Assam Polity Notes & Study Materials                                                                Go To Assam Polity Quiz MCQ

APSC Recruitment 2024 – 18 posts of Motor Vehicle Inspector (MVI) in Transport Department, Assam | Online Apply

APSC Recruitment 2024 – 18 posts of Motor Vehicle Inspector (MVI) in Transport Department, Assam | Online Apply

Jobs vacancy recruitment assam exam

Go to Assam Govt Jobs 

Assam Public Service Commission has released notification for the recruitment of 18 posts of Motor Vehicle Inspector (MVI) in Transport Department, Assam. Interested and eligible candidates may apply online.

 
Category-wise vacancy: 
# UR: 12 posts
# OBC/ MOBC: 4 posts
# SC: 2 posts

 
Scale of Pay: Rs. 22,000/- to 97,000/-, Grade Pay: Rs 9,400/-
 
 
Age Limit: Not be less than 21 years of age and not more than 38 years of age as on 01/01/2024.
 
 
Eligibility criteria for 18 posts of Motor Vehicle Inspector (MVI) in Transport Department, Assam:

Educational Qualification: H.S.L.C./H.S.S.L.C. with 3 (three) years Diploma in Automobile or Mechanical Engineering from a recognised Institute of Govt. of Assam/ Govt. of India and also duly recognised by AICTE.

Citizenship: Candidates must be Indian Citizens as defined in Articles 5 to 8 of the Constitution of India.
 
Residentship:

(i) The candidate must be a permanent resident of Assam.
(ii) The candidate must produce PRC issued in Assam for educational purpose/ Employment Exchange Registration certificate as proof of residency along with the application form.  
 
Age Relaxation: The category-wise age relaxation is as mentioned below.
# SC: 5 years
# ST: 5 years
# OBC/ MOBC: 3 years
# PwBD: 10 years
 
 
Application Fee: Category-wise application fee is as under. 
 
Category
Fee (in Rs.)
General/EWS Rs.297.20/-
SC/ST/OBC/MOBC Rs.197.20/-
BPL/ PwBD Rs.47.20/-
 
 
How to Apply for 18 posts of Motor Vehicle Inspector (MVI) in Transport Department, Assam:
 
Applicants to apply online through APSC’s recruitment website https://apscrecruitment.in/ and register by clicking on ‘Register Here’.
 
 
 
Start & Last Dates for 18 posts of Motor Vehicle Inspector (MVI) in Transport Department, Assam:

Start date of online application: 18 March, 2024

Last date of online application: 17 April, 2024
 
 
Direct Recruitment Process for 18 posts of Motor Vehicle Inspector (MVI) in Transport Department, Assam:

The selection procedure will be notified later on followed by issuing corrigendum/ addendum, if required. The Commission will decide the provision for selection in respect of any post/posts or service/ services considering the status, cadre and grade or the number of applications received for the advertised post/posts or service/ services.
 
The Commission will also decide to conduct examination/test etc zonewise depending upon the numbers of candidates against the districts shown below:
 
Exam Centers:
Sl. No.                   Name of Zones          Districts covered under the Zones

1                             Silchar                         Hailakandi, Karimganj & Cachar

2                             Jorhat                        Golaghat, Dibrugarh, Jorhat, Majuli, Charaideo, Sivasagar & Tinsukia

3                             Nagaon                     Dima Hasao, Hojai, Nagaon, Morigaon, Karbi Anglong & West Karbi Anglong

4                             Tezpur                       Biswanath, Dhemaji, Lakhimpur, Sonitpur & Udalguri

5                             Kokrajhar                  Bongaigaon, Chirang, Dhubri, Goalpara, Kokrajhar & South Salmara

6                             Guwahati                  Barpeta, Baksa, Kamrup, Kamrup (Metro), Nalbari, Darrang, Bajali & Tamulpur.


Important Web-Links for 18 posts of Motor Vehicle Inspector (MVI) in Transport Department, Assam:
 
Online Application Form Click Here
Advertisement Details Click Here
Small Family Norms Declaration
Click Here
Official Website
Click Here
 
 
 
 
 

APSC Recruitment 2024 – 16 posts of Plant Managers in Dairy Development | Online Apply

APSC Recruitment 2024 – 16 posts of Plant Managers in Dairy Development | Online Apply

Jobs vacancy recruitment assam exam

Go to Assam Govt Jobs 

Assam Public Service Commission has released notification for the recruitment of 16 posts of Plant Manager and its equivalent Post viz. Chilling Plant Supervisor/ Milk Tester/ Asstt. Rural Dairy Extension Office (ARDEO)/ Asstt. Distribution Officer (ADO) and its equivalent Posts under Dairy Development Department, Assam (Class-I, Class-B Junior Grade). Interested and eligible candidates may apply online.

 
Category-wise vacancy: 
# UR: 9 posts
# OBC/ MOBC: 4 posts
# SC: 1 post
# STP: 1 posts
# STH: 1 post

 
Scale of Pay: Rs. 30,000/- to 1,10,000/-, Grade Pay: Rs 12,700/-
 
 
Age Limit: Not be less than 21 years of age and not more than 38 years of age as on 01/01/2024.
 
 
Eligibility criteria for 16 posts of Plant Managers in Dairy Development:

Educational Qualification: A candidate must have at least a Degree in Dairy Technology from any recognized university/ Institution.

Citizenship: Candidates must be Indian Citizens as defined in Articles 5 to 8 of the Constitution of India.
 
Residentship:

(i) The candidate must be a permanent resident of Assam.
(ii) The candidate must produce PRC issued in Assam for educational purpose/ Employment Exchange Registration certificate as proof of residency along with the application form.  
 
Age Relaxation: The category-wise age relaxation is as mentioned below.
# SC: 5 years
# ST: 5 years
# OBC/ MOBC: 3 years
# PwBD: 10 years
 
 
Application Fee: Category-wise application fee is as under. 
 
Category
Fee (in Rs.)
General/EWS Rs.297.20/-
SC/ST/OBC/MOBC Rs.197.20/-
BPL/ PwBD Rs.47.20/-
 
 
How to Apply for 16 posts of Plant Managers in Dairy Development:
 
Applicants to apply online through APSC’s recruitment website https://apscrecruitment.in/ and register by clicking on ‘Register Here’.
 
 
 
Start & Last Dates for 16 posts of Plant Managers in Dairy Development:

Start date of online application: 6 March, 2024

Last date of online application: 5 April, 2024
 
 
Direct Recruitment Process for 16 posts of Plant Managers in Dairy Development:

The selection procedure will be notified later on followed by issuing corrigendum/ addendum, if required. The Commission will decide the provision for selection in respect of any post/posts or service/ services considering the status, cadre and grade or the number of applications received for the advertised post/posts or service/ services.
 
The Commission will also decide to conduct examination/test etc zonewise depending upon the numbers of candidates against the districts shown below:
 
Exam Centers:
Sl. No.                   Name of Zones          Districts covered under the Zones

1                             Silchar                         Hailakandi, Karimganj & Cachar

2                             Jorhat                        Golaghat, Dibrugarh, Jorhat, Majuli, Charaideo, Sivasagar & Tinsukia

3                             Nagaon                     Dima Hasao, Hojai, Nagaon, Morigaon, Karbi Anglong & West Karbi Anglong

4                             Tezpur                       Biswanath, Dhemaji, Lakhimpur, Sonitpur & Udalguri

5                             Kokrajhar                  Bongaigaon, Chirang, Dhubri, Goalpara, Kokrajhar & South Salmara

6                             Guwahati                  Barpeta, Baksa, Kamrup, Kamrup (Metro), Nalbari, Darrang, Bajali & Tamulpur.


Important Web-Links for 16 posts of Plant Managers in Dairy Development:
 
Online Application Form Click Here
Advertisement Details Click Here
Small Family Norms Declaration
Click Here
Official Website
Click Here
 
 
 
 
 

APSC Recruitment 2024 – 71 posts of Veterinary Officer/ Block Veterinary Officer | Online Apply

APSC Recruitment 2024 – 71 posts of Veterinary Officer/ Block Veterinary Officer | Online Apply

Jobs vacancy recruitment assam exam

Go to Assam Govt Jobs 

Assam Public Service Commission has released notification for the recruitment of 71 posts of Veterinary Officer/ Block Veterinary Officer, Class-B, Class I (Jr. Grade) under Animal Husbandry & Veterinary Department, Assam, Guwahati. Interested and eligible candidates may apply online.

 
Category-wise vacancy: 
# UR: 37 posts
# OBC/ MOBC: 19 posts
# SC: 5 post
# STP: 7 posts
# STH: 3 post

 
Scale of Pay: Pay Band – 4, Pay scale: Rs. 30,000/- to 1,10,000/-, Grade Pay: Rs 12,700/-
 
 
Age Limit: Not be less than 21 years of age and not more than 38 years of age as on 01/01/2024.
 
 
Eligibility criteria for 71 posts of Veterinary Officer/ Block Veterinary Officer:

Educational Qualification: A candidate must have at least a Degree in Animal Husbandry & Veterinary Science from any recognized university/ Institution.

Citizenship: Candidates must be Indian Citizens as defined in Articles 5 to 8 of the Constitution of India.
 
Residentship:

(i) The candidate must be a permanent resident of Assam.
(ii) The candidate must produce PRC issued in Assam for educational purpose/ Employment Exchange Registration certificate as proof of residency along with the application form.  
 
Age Relaxation: The category-wise age relaxation is as mentioned below.
# SC: 5 years
# ST: 5 years
# OBC/ MOBC: 3 years
# PwBD: 10 years
 
 
Application Fee: Category-wise application fee is as under. 
 
Category
Fee (in Rs.)
General/EWS Rs.297.20/-
SC/ST/OBC/MOBC Rs.197.20/-
BPL/ PwBD Rs.47.20/-
 
 
How to Apply for 71 posts of Veterinary Officer/ Block Veterinary Officer
 
Applicants to apply online through APSC’s recruitment website https://apscrecruitment.in/ and register by clicking on ‘Register Here’.
 
 
 
Start & Last Dates for 71 posts of Veterinary Officer/ Block Veterinary Officer

Start date of online application: 11 March, 2024

Last date of online application: 10 April, 2024

Last date for payment of application fee: 12 April, 2024
 
 
Direct Recruitment Process for 71 posts of Veterinary Officer/ Block Veterinary Officers

The selection procedure will be notified later on followed by issuing corrigendum/ addendum, if required. The Commission will decide the provision for selection in respect of any post/posts or service/ services considering the status, cadre and grade or the number of applications received for the advertised post/posts or service/ services.
 
The Commission will also decide to conduct examination/test etc zonewis depending upon the numbers of candidates against the districts shown below:
 
Exam Centers:
Sl. No.                   Name of Zones          Districts covered under the Zones

1                             Silchar                         Hailakandi, Karimganj & Cachar

2                             Jorhat                        Golaghat, Dibrugarh, Jorhat, Majuli, Charaideo, Sivasagar & Tinsukia

3                             Nagaon                     Dima Hasao, Hojai, Nagaon, Morigaon, Karbi Anglong & West Karbi Anglong

4                             Tezpur                       Biswanath, Dhemaji, Lakhimpur, Sonitpur & Udalguri

5                             Kokrajhar                  Bongaigaon, Chirang, Dhubri, Goalpara, Kokrajhar & South Salmara

6                             Guwahati                  Barpeta, Baksa, Kamrup, Kamrup (Metro), Nalbari, Darrang, Bajali & Tamulpur.


Important Web-Links for 71 posts of Veterinary Officer/ Block Veterinary Officer:
 
Online Application Form Click Here
Advertisement Details Click Here
Small Family Norms Declaration
Click Here
Official Website
Click Here
 
 
 
 
 

APSC Prelims 2023 – General Studies (GS) Paper 1 Solved | Download PDF

APSC Prelims 2023 – General Studies (GS) Paper 1 Solved | Download PDF

APSC Prelim 2023 GS-I Paper – Download PDF

Previous Years APSC Prelim Question Papers (GS & CSAT)

APSC Prelim 2023 General Studies (GS-I) Paper – Download PDF

Go to APSC Prelim Previous Years Questions

Q1. Chief Minister’s Atmanirbhar Asom Abhijan Portal was launched on 23rd September, 2023. To qualify for financial assistance of Rs. 5.0 lakh under the scheme, which of the following criteria are necessary?

  1. The applicant must be a permanent resident of Assam.
  2. The applicant must not be older than 28 years of age .
  3. The applicant must have a bank account opened before 31st December, 2023.
  4. The applicant must have a professional degree in engineering/medical/dental/veterinary/fishery/agriculture, etc. 

Select the correct answer using the codes given below.

  1. I and II only
  2. II and III only
  3. I, III and IV only
  4. I and IV only

 

C. I, III and IV only

 

Q2. Which of the following is/are correct about a Bad Bank?

  1. A bank that is insolvent. 
  2. A bank that has become bankrupt.
  3. A bank whose NPAs have crossed the tolerable limit
  4. A bank set up to buy bad loans and other illiquid holdings of another financial institution. 

Select the correct answer using the codes given below.

  1. I and II only
  2. II and III only
  3. III only
  4. IV only

 

D. IV only

 

Q3. Pick up the wrong statement/statements regarding an interim budget in India.

  1. It provides continuity in governance during the transition period between the governments.
  2. The government cannot propose major taxes or policy reforms in the Budget.
  3. The Economic Survey is presented one day before the presentation of the Interim Budget.
  4. It is an authorization for incurring specific expenditure necessary until a new government is formed. 

Select the correct answer using the codes given below.

  1. I only
  2. I and II
  3. III only
  4. II and IV only

(Our APSC Prelim Test Series 2023 – Test 5 ->Q24. partially matched)

C. III only

 

Q4. INS Imphal was declared to the Indian Navy on 20th October, 2023. Which of the following statements about INS Imphal is/are not correct?

  1. It is a Visakhapatnam-class stealth guided missile destroyer of the Indian Navy.
  2. It was constructed by the Mazagon Dock Shipbuilders Ltd. 
  3. It was Commissioned into the Indian Navy  in December 2023.
  4. The ship boasts 100% indigenous content. 

Select the correct answer using the codes given below.

  1. I only
  2. II and III only
  3. III and IV only
  4. IV only

D. IV only

(Topic covered in our Current Affairs 2023 Module,  part of APSC Prelim Test Series 2023)

 

Q5. The ‘HUMP WWII Museum’ was opened in Arunachal Pradesh in 2023. Which of the following statements are correct pertaining to the Museum?

  1. The Museum has been set up in the town of Pasighat. 
  2. The Hump refers to the dangerous aerial route over the ice covered eastern Himalayan mountains. 
  3. The dangerous route was used to supply British troops battling Japanese Forces after they cut off the overland route through Burma (Myanmar).
  4. The route also earned the moniker ‘Aluminium Trail’ Because of the number of crashes. 

Select the correct answer using the codes given below.

  1. I and II only
  2. I and IV only
  3. I, II and III only
  4. I, II and IV only

D. I, II and IV only

 

Q6. Which of the following lists is correct carrying the names of the five-member bench of the Supreme Court that passed the Ayodhya Verdict on 9th November, 2029? 

  1. Justice Ranjan Gogoi, Justice Ashok Bhushan, Justice Sharad A. Bobde, Justice D. Y. Chandrachud, Justice S. Abdul Nazeer
  2. Justice Ranjan Gogoi, Justice Ashok Bhushan, Justice Sharad A. Bobde, Justice D. Y. Chandrachud, Justice U. U. Lalit
  3. Justice Ranjan Gogoi, Justice Ashok Bhushan, Justice Sharad A. Bobde, Justice D. Y. Chandrachud, Justice Surya Kant
  4. Justice Ranjan Gogoi, Justice Mohammad Hidayatullah, Justice Sharad A.  Bobde, Justice D. Y. Chandrachud, Justice S. Abdul Nazeer

A. Justice Ranjan Gogoi flanked by (L-R) Justice Ashok Bhushan, Justice Sharad Arvind Bobde, Justice Dhananjaya Y Chandrachud, Justice S Abdul Nazeer


Q7. Match following Wildlife Sanctuaries (WLS) of Assam (List-I) with the districts where these are located (List-II)

List-I                                                             List-II

  1. Bordoibam Bilmulkh WLS          1. Lakhimpur
  2. Pabha WLS                                      2. Dhemaji
  3. Pani Dining WLS                            3. Sivasagar
  4. Gibbon WLS                                    4. Jorhat

Select the correct answer using the codes given below.

C. a2, b1, c3, d4

 

Q8. Consider the following statements regarding the 2024 Grammy Awards:

(iI Tabla maestro Zakir Hussain won 3 Grammys

(ii) Flautist Rakesh Chaurasia won 2 Grammys

(iii) Singer Shankar Mahadevan, violinist L. Subramaniam won 1 Grammy each

(iv) The fusion group ‘Shakti’ won the 2024 Grammy Award for the best global music album for ‘This Moment’.

Select the correct statement(s) using the codes given below.

  1. (i) and (ii) only
  2. (iv) only
  3. (i), (ii) and (iv) only
  4. (i), (ii), (iii) and (iv)

(Our APSC Prelim Test Series 2023 – Test 12 ->Q86. matched)

C. (i), (ii) and (iv) only

Q9. Consider the following pairs:

(i) Mahamaya Temple                          : Dhubri

(ii) Dhekiakhowa Bornamghar         : Golaghat

(iii) Malini Than                                      : Dhemaji

(iv) Deopani Temple                            :Jorhat

Select the correctly matched pairs using the codes given below.

B. (i) and (iii) only

 

Q10. Match following personalities (List-I) with the awards/recognitions they has been conferred with (List-II):

List-I                                                    List-II

  1. Hemoprova Chutia            1. Awarded Padma Shri in 2023
  2. Jintimoni Nakul Kalita      2. First female participant from Assam in the T20 Asia Cup hosted in Hong Kung
  3. Uma Chetry                         3. Indian cricket player from Assam in the Asian Games held in Hangzhou, China
  4. Pratima Nandi Narzary    4. Winner of Sahitya Akademi Bal Puraskar 2023

Select the correct answer using the codes given below.

A. A1, b2, c3, d4

(Our APSC Prelim Test Series 2023 – Test 8 ->Q18. matched)
(Topic covered in our Current Affairs 2023 Module,  part of APSC Prelim Test Series 2023)

 

Q11. India breached the 100-medal mark for the first time in the Asian Games at Hangzhou in China in 2023. Select the correct pair of gold and silver medals from the options given below:

  1.  Gold-24, Silver-34
  2.  Gold-28, Silver-38
  3. Gold-28, Silver-32
  4. Gold-22, Silver-38

B. Gold-28, Silver-38

India’s Achievement in 19th Asian Games 2023 – 107 Medals & Records

 

Q12. The operation to evacuate the Indian citizens from Israel during the 2023 Israel-Hamas war is names as

  1. Operation Rahat
  2. Operation Ajay
  3. Operation Devi Shakti
  4. Operation Kaveri

(Our APSC Prelim Test Series 2023 – Test 12 ->Q71 matched)

B. Operation Ajay

 

Q13. Match the following historical/tourist places of Assam (List-I) with the corresponding districts where these are located (List-II):

List-I                                                    List-II

  1. Ghuguha Dol                                1. Darrang
  2. Patharughat                                  2. Dhemaji
  3. Sankhadevi Archeological Site 3. Nagaon
  4. Baduli Khurung                            4. Hojai

(B) a2, b1, c4, d3

Patharughatar Rann of 1894 or Patharughat massacre (History of Assam)

 

Q14. Read the following statements regarding ‘Blockchain’. Which of the following statements is/are correct?

(i) It is a method of recording information that makes it impossible or difficult for the system to be changed, hacked, or manipulated.

(ii) This method can ensure secure transactions, lower compliance expenses and accurate data transfer processing.

(iii) This method is used only in financial service industry.

(iv) Fuji Nakamon is the name used by the presumed pseudonymous person or persons who developed Bitcoin.

Select the correct answer using the codes given below.

  1. (i) only
  2. (i) and (ii) only
  3. (i), (i) and (iii) only
  4. (i), (ii) and (iv) only

( Daily Current Affairs Qs in Facebook page: Adhikary Education | Silchar (facebook.com) matched)

B. (i) and (ii) only

 

Q15. Who among the following was awarded the Siu-Ka-Pha Award, 2023?

(A) Dhirendra Nath Bezbaruah

(B) Pranab Jyoti Deka

(C) Nandeswar Daimari

(D) Ranjan Gogoi

 

(A) Dhirendra Nath Bezbaruah

 

Q16. Who among the following are Arjuna Award winners from Assam?

(i) Bhogeswar Baruah

(ii) Monalisa Baruah Mehta

(iii) Amlan Borgohain

(iv) Jayanta Talukdar

Select the correct answer using the codes given below:

(A) (i) and (ii) only

(B) (i), (ii) and (iii) only

(C) (i), (ii) and (iv) only

(D) (i) and (iii) only

 

(C) (i), (ii) and (iv) only

 

Q17. The Mukhyamantn Mahila Udyamita Abhiyan (MMUA) was announced in January 2024. What are the eligibility criteria for the scheme?

(i) All women self-help groups of Assam are eligible for the scheme.

(ii) If the applicants have girl children, they must be enrolled in school

(iii) The trees the applicants planted under the Amrit Briksha Andolan should be surviving.

(iv) General and OBC categories applicants cannot have more than three children.

Select the correct answer using the codes given below.

(A) (i), (ii) and (iii) only

(B) (i), (ii) and (iv) only

(C) (ii) and (iv) only

(D) (ii), (iii) and (iv) only

(Our APSC Prelim Test Series 2023 – Test 5 ->Q1 partially matched)

(D) (ii), (iii) and (iv) only

 

Q18. If an individual is awarded with a gold medal called Lachit Borphukan Medal as the best Army Cadet, where has he/she passed out from?

(A) Indian Military Academy (IMA)

(B) National Defence Academy (NDA)

(C) Rashtriya Indian Military College (RIMC)

(D) Army Cadet College (ACC)

 

(B) National Defence Academy (NDA)

 

Q19. Arrange the following Speakers of the Assam Legislative Assembly in the chronological order:

(i) Pulakesh Baruah

(ii) Kuladhar Chaliha

(iii) Debeswar Sarma

(iv) Mahendra Mohan Choudhury


Select the correct answer using the codes given below.

(A) (iii)-(iv)-(ii)-{i)

(B) (iv)-(iii)-(ii)-(i)

(C) (iii)-(ii)-(iv)-(i)

(D) (iv)-(iii)-(i)-(ii)

 

(C) (iii)-(ii)-(iv)-(i)

 

Q20. Arrange, in descending order, the population in Assam by languages as per the Census of India, 2011.

(A) Assamese, Mising, Bengali, Bodo

(B) Assamese, Bengali, Bodo, Mising

(C) Assamese, Bodo, Mising, Bengali

(D) Assamese, Mising, Bodo, Bengali

 

(B) Assamese, Bengali, Bodo, Mising

Assam Demography in Census 2011 – Population, Sex Ratio, Literacy rate Highlights & figures

Q21. Which of the following is/are NOT the work/works of Madhavdev?

(i) Nam Ghosa

(ii) Bhakti Ratnavali

(iii) Bhumi Letowa

(iv) Balichalan

Select the correct answer using the codes given below.

  1. (i) only
  2. (ii) and (iv) only
  3. (iv) only
  4. (iii) and (iv) only

 

C. (iv) only

 

Q22. Match the following places of Assam (List-I) with the districts where these are located (List-II):

List-I                            List-II

  1. Titabar           1. Sivasagar
  2. Doulasal        2. Barpeta
  3. Nazira            3. Jorhat
  4. Sarthebari    4. Nalbari

Select the correct answer using the codes given below.

(D) a3, b4, c1, d2

 

Q23. Which of the following names of the Agroclimatic Zones of Assam is/are NOT correct?

(i) Upper Brahmaputra Valley

(ii) North Central Brahmaputra Valley

(iii) Barail Zone

(iv) Hill Zone

Select the correct answer using the codes given below.

(A) {i) only

(B) iii) and {iv) only

(C) {ii) only

(D) (ii) and (iii) only

 

(D) (ii) and (iii) only

 

Q24. Which of the following statements are correct about the Women’s Reservation Bill, 2023?

(i) The Women’s Reservation Bill, 2023 proposes 33% reservation for women in the Lok Sabha and the Assemblies of States and National Capital Territory of Delhi.

(ii) Similar reservation will also be provided within the seats reserved for SC and ST.

(ii) The Bill proposes that the reservation will continue for 25 years.

(iv) The Bill which reserves 33% of seats for women in the Lok Sabha and State Assemblies includes the quota for women from the OBCs.

Select the correct answer using the codes given below.

(A) (i) and (ii) only

(B) (i) and [iii) only

(C) (i), (ii) and (iii) only

(D) (i), (i), (iii) and (iv)

 

(A) (i) and (ii) only

 

Q25. Match the following schemes (List-I) with the correct objectives (List-II):

List-I                                                    List-II

a. Beti Bachao, Beti Padhao 1. It gives important to the nutritional level of adolescent girls, pregnant women, lactating mothers and children of 0-6 years of age.

b. Baalika Samriddhi Yojana 2. It offers free course materials/online resources such as video study materials, class materials for girl students in XI and XII standards.

c. CBSE Udaan Scheme 3. To provide fiscal support to young girls and their mothers who belong to BPL category

d. Poshan Abhiyan 4. Removing or preventing gender bias selective abortions for the girl child

 

(A) a 4             b 3       c 2       d 1

 

Q26. Match the following Particularly Vulnerable Tribal Groups (PVTGs) (List-l) with the States/Union Territories they belong to (List-Il) ;

List—1                                     List—II

a. Maram 1. Andaman and Nicobar Islands

b. Jarawa 2. Tripura

c. Birhor 3. Manipur

d. Reang 4. Odisha

Select the correct answer using the codes given below.

 

(B)       a 3         b 1          c 4          d 2

 

Q27. Which of the following statements are correct about the Scheduled Castes in Assam as per the 2011 Census?

(i) Scheduled Caste population constitutes about 7-15% of the total population of Assam.

(ii) 16 sub-castes are recognized as SCs in Assam, such as Basfor, Bhuimali, Bania, Dhupi, Dholi, Hira, Kaibartta, etc.

(iii) Nagaon district has the highest percentage of SC population followed by Cachar and Kamrup.

(iv) The literacy rate of SCs in Assam is 66°76% with SC females constituting 55° and SC males constituting 44-48%.

Select the correct answer using the codes given below.

(A) (i) and (ii) only

(B) (i), (ii) and (iii) only

(C) (iii) and (iv) only

(D) (i, ii), (iii) and (iv)

 

(B) (i), (ii) and (iii) only

 

Q28. Match the following demographic terms (List-1) with their meanings (definitions) (List-Il) :

List-I                                                    List-II

a. Birthrate                          1. Number of live births per woman who were born in a particular year

b. Crude birth rate            2. Number of live births rate Per one  thousand inhabitants in a year

c. Cohort fertility               3. Determined not only by the fertility of a given population but also by its age structure

d. Birth deficit                    4. When the number of live births is lower than the number of deaths within a defined period of time

Select the correct answer using the codes given below.

 

(A) a 2             b 3       c 1       d 4

 

Q29. Match the following personalities (List-}) with the awards conferred on them (List-Hl):

List—I                                   List—Il

  1. Pranab Jyoti Deka   1. Sahitya Akademi Award
  2. Arup Kumar Dutta  2. Kalicharan Brahma Award
  3. Dr. Joyanti Chutia    3. Shri Madhavdev Award
  4. Dr. Tiyathi Zameer   4. Sati Sadhani Award

Select the correct answer using the codes given below.

B. a 1       b 3       c 4       d 2

 

(Our APSC Prelim Test Series 2023 – Test 10 ->Q62 matched)

 

Q30. The Ramchandi Temple, which was built during 12th century, is situated on the bank of which of the rivers of Assam?

(A) Gadadhar

(8) Kushiyara

(C) Longai

(D) Mahur

 

(D) Mahur

Q31. Consider the following statements about Sardar Vallabhbhai Patel:

(i) He got the moniker ‘Sardar’ due to his organizational skills during Bardoli Satyagraha.

(ii) The prison where Mahatma Gandhi and Sardar were cellmates was Naini Prison.

(iii) He was awarded the Bharat Ratna in 1989,

(iv) He was the first Deputy Prime Minister of India.

Select the incorrect statements using the codes given below.

(A) (i) and (ii) only

(B) (ii) and (iii) only

(C) (iii) and (iv) only

(D) (i) and (iv) only

 

(B) (ii) and (iii) only

 

Q32. Match the following names (List-I) with the Ahom rulers (List-II):

List-I                                        List-II

  1. Susenghphaa             1. Rajeswar Singha
  2. Suremphaa                 2. Chakradhwaj Singha
  3. Sutemla                       3. Pratap Singha
  4. Supangmung            4. Jayadhwaj Singha

Select the correct statements using the codes given below.

 

B. a 3       b 1       c 4       d 2

 

Q33. Gaurinath Singha entered into an Agreement with the East India Company on which of the following terms?

(i) All imports into Assam were to be subjected to a duty of 10%

(ii) The duty on export was to be 10%

(iii) Rice and grains were to be exempted from any duty

(iv) Two custom houses were to be established at Kandahar and Hadira.

Select the correct answers using the codes given below.

(A) (i) and (ii) only

(B) (i), (ii) and (iii) only

(C) (i), II and (iv) only

(D) (i), (ii), (iii) and (iv)

 

(B) (i), (ii) and (iii) only

 

Q34. Match the following authors (List-I) with the books written by them (List-II):

List-I                                                       List-II

  1. Gunabhiram Baruah                1. Asamiya Byakaran
  2. Harakanta Sarma Baruah       2. Asamiya Lorar Mitra
  3. Anandaram Dhekial Phukan 3. Ram Navami
  4. Hemchandra Baruah               4. Sadar Aminar Atmajibani

Select the correct statements using the codes given below.

 

(C) a 3     b 4       c 2       d 1

(Our APSC Prelim Test Series 2023 – Test 8 ->Q17 partially matched)

 

Q35. Match the following associations (List-I) with their respective founders (List-II):

List—I                                              List—II

  1. Indian League                    1. Ananda Mohan Bose
  2. Indian Association            2. Jagannath Baruah
  3. Assam Association            3. Sisir Kumar Ghosh
  4. Jorhat Sarbajanik Sabha 4. Manik Chandra Baruah

Select the correct statements using the codes given below.

 

(B) a 3             b 1       c 4       d 2

 

Q36. The Global Drought Snapshot, released in December 2023 at the beginning of the 28th Conference of the Parties (COP-28) to the UN Framework Convention on Climate Change (UNFCCC) in Dubai, exposes the devastating consequences of droughts on lives, economies, and various sectors. Based on the data reported by 101 parties to the UNCCD, which of the following statements is/are correct?

(i} The report reveals that a staggering 1.84 billion people experienced the effects of drought. Among them 4-7% faced severe or extreme drought conditions.

(ii) Notably South Africa faced the highest number of declarations.

(iii) Drought severity prompted emergency declarations in 23 countries.

(iv) The US, Canada, India and Sri Lanka are the countries that never declared drought emergencies.

Select the correct answer using the codes given below.

(A) (i) only

(B) (ii) and (iv) only

(C) (i) and (iii) only

(D} (i), (ii), (i) and (iv)

 

(C) (i) and (iii) only

 

Q37. Which of the following sessions of Asom Sahitya Sabha was/were presided over by Sitanath Brahma Choudhury?

(i) Tinsukia

(ii) Dipbu

(iii) Sivasagar

(iv) Tezpur

Select the correct answer using the codes given below.

(A) (i) only

(B) (i) and (ii) only

(C) (i) and (iii) only

(D) (iii) and (iv) only

 

(B) (i) and (ii) only

 

Q38. Consider the following statements regarding Pradhan Mantri Matsya Sampada Yojana (PMMSY) :

(i) It is the flagship scheme for focused and sustainable development of fisheries in India.

(ii) It is implemented in all States/ Union Territories as part of Atmanirbhar Bharat package for a period of five years from Financial Year 2020-21 to Financial Year 2024-25

(iii) The Central Govt bears the project cost and States/UT share the cost of sub-components of the scheme.

(iv) Ornamental fish cultivation is kept out of focus under PMMSY.

Select the correct Statement(s) using the codes given below.

(A) (i) only

(B) (i) and (iii) only

(C) (iv) only

(D) (ii) only

 

(C) (iv) only

 

Q39. Consider the following statements regarding Jal Jeevan Mission (JJM) ;

(i) The programme includes recharge and reuse through greywater management, water conservation and rainwater harvesting.

(ii) It is a demand-driven scheme with strong emphasis on Jan Bhagidari (community engagement) as a crucial pillar of the project.

(iii) The United Nations Office for Project Services (UNOPS) is collaborating with the Denmark Government to support the mission in Uttar Pradesh.

(iv) An average of 29 million people will be employed annually during the mission’s construction phase.

Select the correct statement/Statements using the codes given below.

(A) (i) only

(B) (i) and (ii) only

(C) (ii) and (iii) only

(D) (i), (ii), (iii) and (iv)

 

(D) (i), (ii), (iii) and (iv)

 

Q40. Consider the following statements:

(i) MV Ganga Vilas is the world’s longest river cruise covering over 51 days across 2 countries, 5 States and 27 river systems.

(ii) Prime Minister Narendra Modi flagged off the cruise from Varanasi on 13th January, 2023.

(iii) The international cruise being offered  by Antara Luxury River Cruises who is operating since 2009.

(iv) The cruise  culminated the journey on 28th February in Sadiya.

Select the incorrect statements using the codes given below.

(A) (ii) only

(B) (iv) only

(C) (i) and (ii) only

(D) (ii) and (iii) only

 

(B) (iv) only

 

Q41. Which of the following dramas/Ankiya Nats was/were not composed by Sankardev?

  1. Kaliya Daman
  2. Patni Prasada
  3. Keli Gopal
  4. Arjun Bhanjan

 Select the correct answer using the codes given below

(A) (i) and (iii) only

(B) (ii) only

(C) (ii) and (iii) only

(D) (iv) only

 

(D) (iv) only

 

Q42. India celebrated her 75th Republic Day on 26th January, 2024. Which of the following statements is/are not correct about the celebration of the Republic Day, 2024?

  1. The Parade was held on Kartavya Path.
  2. The theme of the 75th Republic Day was ‘Biksit Bharat’ and ‘Bharat ; Loktantra Ki Matruka’,
  3. The three States which presented tableaux from North-East India were Meghalaya, Manipur and Nagaland.
  4. ISRO presented the historic lunar landing of Chandrayaan-IIl at the Parade.

Select the correct answer using the codes given below.

(A) (i) only

(B) (iii) only

(C) (ii) and (iv) only

(D) (i) and (iii) only

(Our APSC Prelim Test Series 2023 – Test 6 ->Q12  matched)
(Topic covered in our Current Affairs 2023 Module,  part of APSC Prelim Test Series 2023)

(B) (iii) only

 

Q43. All the Assam Baibhav, Assam Saurav and Assam Gourav awardees are offered certain benefits/facilities. Which of the following is/are not included in the list of benefits?

(i) They will get free medical treatment in government hospitals and stay in paying cabins, wherever available.

(ii) They will be provided free accommodation in State Government Circuit Houses, Guest Houses and Assam Bhawans.

(iii) ‘They will be regular invitees for all the official functions as distinguished guests.

(iv) The Government of Assam will provide them up to Rs. 5 Lakh for medical expenses towards the treatment of critical diseases.

Select the correct answer using the codes given below.

(A) (ii) and (iii) only

(B) (iii) and (iv) only

(C) (i) and (iii) only

(D) (iv) only

(Our APSC Prelim Test Series 2023 – Test 8 ->Q31  matched)
(Topic covered in our Current Affairs 2023 Module,  part of APSC Prelim Test Series 2023)

(D) (iv) only

 

Q44. Consider the following statements:

(i) NISAR is a low Earth orbit observatory being jointly developed by ISRO and NASA.

(ii) NISAR is a low Earth orbit observatory being jointly developed by ISRO and European Space Agency.

(iii) The objective of NISAR is to co-develop and launch a dual frequency synthetic aperture RADAR on an Earth observation satellite. 

(iv) NISAR is planned to be launched in 2024.

Select the correct answer using the codes given below.

(A) (i) and (iii) only

(B) (ii) and (iii) only

(C) (i) and (iv) only

(D) (i), (iii) and (iv) only

(Our APSC Prelim Test Series 2023 – Test 12 ->Q76  partially matched)

(D) (i), (iii) and (iv) only

 

Q45. Consider the following statements:

(i) India’s solar mission Aditya-L1 has been placed in halo orbit around Sun-Earth Lagrange point 1.

(ii) Lagrange points are positions of equilibrium for a small object under the gravitational pull of two massive orbiting bodies.

(iii) There are 5 Lagrange points for the Sun-Earth system

(iv) There are 5 Lagrange points for the Earth-Moon system also.

Select the correct statement/ statements using the codes given below.

(A) (i) only

(B) (i) and (ii) only

(C) (i), (ii) and (iii) only

(D) (i), (ii), (iii) and (iv)

(Topic covered in our Current Affairs 2023 Module,  part of APSC Prelim Test Series 2023)

(D) (i), (ii), (iii) and (iv)

 

Q46. In the Interim Union Budget, 2024, the Finance Minister mentioned four specific groups of the population to focus on. Which of the following is/are not in the list?

(i) Poor

(ii) Women

(iii) Entrepreneur

(iv) Youth

Select the correct statement/ statements using the codes given below.

(A) (i) only

(B) (ii) and (iv) only

(C) (iii) only

(D) (iii) and (iv) only

 

(C) (iii) only

  

Q47.  Consider the following statements.

(i) In computer parlance, a cookie is a malicius computer virus.

(ii) In computer parlance, a cookie is a small piece of text sent by a website to the internet browser.

(iii) Internet cookies are used by banking websites only.

(iv) Internet cookies help the websites to remember the client’s visit to the website.

Select the correct statement/ statements using the codes given below.

(A) (i) only

(B) (ii) only

(C) (ii) and (iii) only

(D) (ii) and (iv) only

 

(D) (ii) and (iv) only

 

Q48. What is/are the factor/factors that has/have prompted the colonial rulers to initiate jute cultivation in Brahmaputra Valley?

(i) Saturation of land suitable for jute in Bengal

(ii) High demand of jute in the world market

(iii) Abundant labour supply in Brahmaputra Valley.

Select the correct answer using codes given below.

(A) (i) only

(B) (ii) only

(C) (i) and (ii) only

(D) (i), (ii) and (iii)

 

(C) (i) and (ii) only

 

Q49. Which of the following characteristics make a braided river different from a meandering river of similar discharge?

(i) Wider and Shallower

(ii) Transports high sediment load

(iii) Erodes riverbanks rapidly

(iv) Deeper

Select the correct answer using codes given below.

(A) (i), (ii) and (iii) only

(B) (i), (ii) and (iv) only

(C) (i), (iii) and (iv) only

(D) (ii), (iii) and (iv)

 

(A) (i), (ii) and (iii) only

 

Q50. In which of the following Satras in Assam, the religious head (Satradhikar) is elected?

(i) Auniati

(ii) Dakhinpat

(iii) Kamalabari

(iv) Barpeta

Select the correct answer using codes given below.

(A) (i) and (iii) only

(B) (i) and (iii) only

(C) (iii) only

(D) (iv) only

 

 (D) (iv) only

Q51. As per the tripartite Memorandum of settlement signed between Government of India, Government of Assam with representatives of United Liberation Front of Assam (ULFA) on 29th december 2023, which of the following will not be set up in Assam?

  1. Indian Institute of Management
  2. School of planning and Architecture
  3. Indian Institute of Science
  4. Indian Institute of Science Education and Research

Select the correct answer using the codes given below.

  1. I only
  2. II and IV only
  3. III only
  4. III and IV only

 

C. III only

 

Q52. Match the following books (List-I) with their authors (List-II): 

List I                                                                          List II

  1. A Comprehensive History of Assam        1. Arup Jyoti Saikia
  2. The Quest for Modern Assam History     2. Arup Kumar Dutta 
  3. The Ahoms: A Reimagined History         3. Dr Nagen Saikia
  4. Asamiya Manuhar Itihas                            4. Sarnalata Baruah 

         Select the correct answer using the codes given below.

 

D. a4, b1,   c2, d3, 

 

Q53. With reference to the Charaideo Maidams, consider the following statements; 

  1. These are mounds containing the remains of the loyalty of the Varman dynasty.
  2. They are located along the foothills of the Patkai Range
  3. Charaideo Maidams is now a UNESCO World Heritage Site. 
  4. The first king who was buried after death here was Swargadeo Gadadhar Singha.

Select the correct statement/ statements using the codes given below. 

  1. I and III only
  2. I, II and IV only
  3. II only
  4. I, II and III only

(Our APSC Prelim Test Series 2023 – Test 8 ->Q60  partially matched)

C. II only

 

Q54. With reference to the Archeological Survey of India (ASI), consider the following statements:

  1. The prime concern of ASI is the maintenance of ancient monuments and archeological sites and remains of national importance. 
  2. It does not regulate the Antiquities and Art Treasures Act, 1972. 
  3. Alexander Cunningham was appointed as the first Director General of ASI.
  4. ASI is attached to the Ministry of Education, Government of India. 

Select the correct statement/ statements using the codes given below. 

  1. I only
  2. I and III only
  3. III only
  4. I, II, III and IV

 

B. I and III only

 

Q55. Match the following wetlands (List-I) with the districts where these are located (List-II): 

List I                                           List II

  1. Urpad Beel                    1. Bongaigaon
  2. Son Beel                        2. Goalpara
  3. Tamranga Beel            3. Morigaon
  4. Nandini-Sonai Beel    4. Karimganj 

Select the correct answer using the codes given below.

(Our APSC Prelim Test Series 2023 – Test 2 ->Q65  partially matched)

A. a2, b4, c1, d3

 

Q56. Match the following Institution (List-I) with their years of establishment (List-II): 

List I                                                           List II

  1. Cotton College                                1. 1901
  2. Gauhati University                         2. 1969
  3. Assam Agricultural University    3. 1948
  4. Tezpur University                           4. 1994

Select the correct answer using the codes given below.


D. a1, b3, c2, d4

 

Q57. Consider the following developments pertaining to the history of Modern Education in India: 

  1. Sadler University Commission
  2. Lord Macaulay’s minute 
  3. Raleigh Commission
  4. Wood’s Dispatch 

Select the correct chronological order of these developments using the codes given below.

  1. I, III, II, IV
  2. II, III, I, IV
  3. II, IV, III, I
  4. I, II, III, IV


C. II, IV, III, I

 

Q58. With reference to the Khajuraho Group of Monuments, consider the following statements:

  1. The temples at Khajuraho were built during the Chandela dynasty.
  2. The temples at Khajuraho were associated with Hinduism and Jainism.
  3. The temples were built in Dravidian-style architecture.
  4. These are situated in Chhatarpur district in Maharashtra. 

Select the correct statement/ statements using the codes given below. 

  1. I only
  2. I and II only
  3. I, II and III only
  4. I and IV only

B. I and II only

 

Q59. Consider the following events pertaining to the Freedom Movement in India:

  1. Mountbatten Plan 
  2. Quit India Movement
  3. Wavell Plan
  4. Royal Indian Navy Mutiny

Select the correct chronological order of these events using the codes given below.

  1. II, III, IV, I
  2. II, I, III, IV
  3. II, IV, I, III
  4. II, I, IV, III

(Our APSC Prelim Test Series 2023 – Test 1 ->Q3  explanation)

A. II, III, IV, I

 

Q60. The NITI Forum for North-East was set up in February 2018 to address various challenges in the region and recommend requisite interventions to achieve sustainable economic growth. The forum has identified 5 focus areas for the North East. Which of the following is/are not in the list of focus areas?

  1. Tourism
  2. Pisciculture
  3. Pulses
  4. Fruits

Select the correct answer using the codes given below.

  1. I only
  2. I  and II only
  3. III only
  4. III and IV only

D. III and IV only

Q61. Which of the following pairs is/are incorrectly matched:

  1. State bird of Assam           : Hornbill
  2. State animal of Assam       : Rhino
  3. State flower of Assam        : Sunflower
  4. State tree of Assam           : Hollong

Select the correct answer using the codes given below.

  1. I only
  2. I and III only
  3. II and III only
  4. III and IV only

(Our APSC Prelim Test Series 2023 – Test 2 ->Q33  explanation)

B. I and III only

 

Q62. Consider the following pairs:

Places                     Events in the life of Buddha

  1. Lumbini       : Birth
  2. Sarnath        : Dharmachakrapavartana 
  3. Kushinagar : Enlightenment 
  4. Benaras       :  The Miracles 

Select the correctly matched pairs using the codes given below.

  1. I and IV only
  2. I and II only
  3. I and III only
  4. I, II and III only 

(Our APSC Prelim Test Series 2023 – Test 1 ->Q66 & Q68)

B. I and II only

 

Q63. Consider the following statements regarding the Delimitation Commission of India:

  1. The Delimitation Commission is appointed by the President of India and works in collaboration with the President of India. 
  2. Article 82 of the Constitution of India states that the Parliament is required by law to enact a Delimitation Act following each Census.
  3. The first Delimitation exercise in post-independent India was held in 1952 as per 1951 Census. 
  4. The present delimitation of parliamentary constituencies has been done on the basis of the 2001 Census figures under the provisions of the Delimitation Act, 2002

Select the correct statement/ statements using the codes given below. 

  1. I only
  2. I and II only
  3. I, II and III only
  4. I, II, III and IV

(Our APSC Prelim Test Series 2023 – Test 4 ->Q66)

D. I, II, III and IV

 

Q64. Consider the following statements regarding the Election Commission of India: 

  1. The Election Commission of India is a body constitutionally empowered to conduct free and fair elections to the National and State Legislatures including the Panchayats. 
  2. The power of the Election Commission of India derived is from Article 324 of the Constitution of India.
  3. Rajiv Kumar is the present Chief Election Commissioner, who joined his office on 15th May, 2022. 
  4. The Election Commission is reconstituted every five years. 

Select the incorrect statement/ statements using the codes given below. 

  1. I and II only
  2. II and III only
  3. I and IV only
  4. III only 

(Our APSC Prelim Test Series 2023 – Test 4 ->Q45)

C. I and IV only

 

Q65. Consider the following statements regarding the Goods and Services Tax (GST) of India:

  1. GST is a successor of VAT used in India on the supply of goods and services.
  2. GST came into effect on 1st July, 2017 through the implementation of the one hundred and first Amendment of the Constitution of India.
  3. Every decision of the GST Council is to be taken by a majority of not less than three-fourths of the weighted votes of the members present and voting at the meeting.
  4. The GST Council is chaired by the Prime Minister of India. 

Select the incorrect statement/ statements using the codes given below.

  1. I only
  2. II only
  3. III and IV only
  4. I, II and III only

D. I, II and III only

 

Q66. Consider the following statements regarding Article 226 of the Constitution of India:

  1. Article 226 of the Constitution of India empowers a High Court to issue writs including habeas corpus, mandamus, certiorari, prohibition and quo warranto for the enforcement of the Fundamental Rights of the citizens and for any other purpose. 
  2. The phrase ‘for any other purpose’ refers to the enforcement of an ordinary legal rights. This implies that the writ jurisdiction of the High Court is wider than that of of the Supreme Court.
  3. The Supreme Courts can issue writs only for the enforcement of the Fundamental Rights and not for any other purpose, i.e, it does not extend to a case where the breach of an ordinary legal rights is alleged.
  4. The High Courts can issue writs to any person, authority and government not only within its territorial jurisdiction but also outside its territorial jurisdiction if the cause of action arises within its territorial jurisdiction.  

Select the correct statements using the codes given below.

  1. I, II and III only
  2. III and IV only
  3. I and III only
  4. I, II, III and IV

D. I, II, III and IV

 

Q67. Which among the following autonomous councils in Assam have been formed under the sixth schedule of the constitution of India? 

  1. Bodoland Territorial Council or Bodoland Territorial Region
  2. Karbi Anglong Autonomous Council 
  3. Dima Hasao Autonomous Council 
  4. Rabha Hasong Autonomous Council

Select the correct answer using the codes given below.

  1. I only
  2. I and II only
  3. I, II and III only
  4. I, II, III and IV

(Our APSC Prelim Test Series 2023 – Test 8 ->Q67)

C. I, II and III only

 

Q68. Consider the following statements regarding the special provisions to some of the States of India made by the Constitution:

  1. Article 371C provides for the special provisions for the state of Assam. Under this article the President of India is empowered to provide for the creation of committee of the Assam legislative Assembly consisting of members elected from Tribal Areas of the State and such other members as he/she may specify.
  2. Article 371C was added by the 22nd Amendment Act of 1969.
  3. Article 371D deals with special provisions with respect to the State of Andhra Pradesh and Article 371E deals with the establishments of Central University in Andhra Pradesh. 
  4. 32nd Constitutional Amendment Act, which was made in 1973, inserted Article 371D and 371E into the Constitution of India. 

Select the correct statement/statements using the codes given below.

  1. I only
  2. III and IV only
  3. I and II only
  4. I and III only


C. I and II only

 

Q69. Consider the following statements regarding the provisions of official languages in India:

  1. Article 343 of the Constitution of India has made Hindi in Devanagari as the National Language of India.
  2. The Legislature of a State may by law adopt any one or more of the languages in use in the state as the official language/languages of the State as per Article 345 of the Constitution of India.
  3. The Assam Official Language Act, 1960 has been legislated as per Article 345 of the Constitution of India.

Select the correct statements using the codes given below.

  1. I and II only
  2. II and III only
  3. I and III only
  4. I, II and III

B. II and III only

 

Q70. Match the following schedules of Indian Constitution (List-I) with their subject matters in (List- II): 

List-I                                          List-II

  1. Fourth Schedule           1. Forms of oaths and affirmations 
  2. Eleventh Schedule       2. Languages
  3. Eighth Schedule           3. Power of Panchayats 
  4. Third Schedule              4. Allocations of Seats in the Council of States

Select the correct answer using the codes given below.

(Our APSC Prelim Test Series 2023 – Test 12 ->Q2)

B. a4, b3, c2. d1

Q71. Consider the following factors regarding the declaration of a Political Party as a State Party by the Election Commission of India: 

  1. If it secures 6% of the valid votes polled in the states in a general election to the respective State Legislative Assembly, and also it wins 2 seats in the same State Legislative Assembly 
  2. If it secures 6% of the total votes in the State in a general election to the Lok Sabha and also it wins 1 seat in the Lok sabha from the same state
  3. If it wins 3% of the seats in the Legislative Assembly at a general election to the Legislative Assembly of the state concerned for 3 seats in the Assembly (whichever is more)
  4. If it secures 16% of the total valid votes polled in the State at a general election to the Lok Sabha from the State or to the State Legislative Assembly

Select the incorrect factors/factor using the codes given below.

  1. I, II and III only
  2. III and IV only
  3. I and III only
  4. IV only

(Our APSC Prelim Test Series 2023 – Test 4 ->Q49 partially matched)

D. IV only

 

Q72. Which of the following statements are correct about the difference between Censure and no-confidence motion?

  1. A censure motion should state the reasons for its adoption. On the other hand a no-confidence motion need not state the reasons for its adoption. 
  2. A no-confidence motion can be moved only against the council of ministers. In contrast a censure motion can be moved against the council of ministers or an individual minister or a group of ministers.
  3. The Government must resign if a no confidence motion is passed. On the other hand the Government need not resign if a censure motion is passed. 
  4. A censure motion can be moved in both the Lok Sabha and the Rajya Sabha, whereas a no-confidence motion can be moved only in the Lok Sabha.

Select the correct answer using the codes given below. 

  1. I, III and IV only
  2. II, III and IV only
  3. I and II only
  4. I, II and III only


D. I, II and III only

 

Q73. Consider the following statements regarding the Assam legislative Assembly:

  1. According to the provisions of the Government of India Act 1935, a Bicameral Legislature of Assam Province came into existence in 1937.
  2. The first sitting of the Assam Legislative Assembly took place on April 7, 1937 in the Assembly Chamber at Shillong.
  3. Pulakesh Barua and Ganesh Kutum were the Speakers of Assam Legislative Assembly from Asom Gana Parishad and Janata Party respectively. 
  4. Babu Basanta Kumar Das was the first Speaker of the Assam Legislative Assembly.

Select the correct statement/ statements using the codes given below. 

  1. I only
  2. II and III only
  3. I, III and IV only
  4. I, II, III and IV

(Our APSC Prelim Test Series 2023 – Test 4 ->Q1)

C. I, III and IV only

 

Q74. It is easier to roll barrel than to pull it because

  1. the full weight of the barrel comes into play when it is pulled.
  2. rolling friction is much less than sliding friction. 
  3. the surface area of the barrel in contact with the road is more in case of pulling.
  4. of a reason other than those mentioned


B. rolling friction is much less than sliding friction. 

 

Q75. The mass of a body is different from its weight because 

  1. Mass is a variable quantity whereas weight is constant.
  2. Mass varies very little at different places whereas weight varies a lot
  3. Mass is constant but weight decreases as the body moves from the Poles to the Equator
  4. Mass is a measure of quantity of matter whereas weight is a force

Select the correct reason/ reasons using the codes given below. 

  1. I only
  2. II and IV
  3. III only
  4. III and IV

B. II and IV

 

Q76. Paddy fields are chief sources of which of the following greenhouse gasses?

  1. Methane
  2. Carbon Dioxide
  3. Nitrous Oxide
  4. Ozone 

Select the correct answer using the codes given below.

  1. I only
  2. II only
  3. I and III
  4. II, III and IV 

A. I only

 

Q77. Which of the following groups contains/contain only non-biodegradable materials?

  1. Wood, paper, leather 
  2. Plastic, bakelite, DDT
  3. Plastic, thermocol, wood
  4. Thermocol, plates made from dried leaves, detergents

Select the correct answer using the codes given below. 

  1. I and II only
  2. II only
  3. II and III only
  4. III and IV only

B. II only

 

Q78. For a given liquid, the rate of evaporation depends on

  1. The temperature of the liquid
  2. The temperature of air
  3. The surface area of the liquid

Select the incorrect factor/factors using the codes given below.

  1. I only
  2. I and II only
  3. II and III only
  4. I, II and III


D. I, II and III

 

Q79. One should not connect a number of electrical appliances to the same power socket because 

  1. This can damage the appliances 
  2. This can damage the domestic wiring due to overheating
  3. The appliances will not get full voltage

Select the valid reason/reasons using the codes given below.

  1. II and III only
  2. II only
  3. I and II
  4. III only

B. II only

 

Q80. Consider the following pairs of scientist and their discoveries:

Scientists                  Discoveries

  1. Copernicus          Law of gravitation       
  2. Kepler                  Law of planetary motion
  3. Galileo                 Moons of Jupiter
  4. Newton                 Heliocentric theory

Select the correctly matched pairs using the codes given below:

  1. I and II only
  2. I, II and III only
  3. I, II, III and IV
  4. II and III

D. II and III

Q81.  From the distance- time graphs of two motorists A and B, it can be seen that

  1. A travelled with uniform speed but B did not.
  2. A travelled faster than B throughout
  3. B travelled faster than A throughout
  4. A and B travelled with equal speed.

C. B travelled faster than A throughout

 

Q82. Solutions are of many types. Consider the following types of solution with an example of each. 

  1. Gas in liquid  – Vinegar
  2. Solid in liquid – Brine
  3. Solid in solid  – Brass
  4. Liquid in liquid – Coca-Cola

Select the correctly matched pairs using the codes given below:

  1. I and II only
  2. II and III only
  3. I, III and IV only
  4. II, III and IV only


B. II and III only

 

Q83. Which of the following elements does sugar contain?

  1. Carbon
  2. Nitrogen
  3. Hydrogen
  4. Oxygen

Select the correct answer using the codes given below. 

  1. I, III and IV only
  2. I, II and III only
  3. I and III
  4. I and IV


A. I, III and IV only

 

Q84. Consider the following statements:

  1. Assam was formed into a Chief Commissioner’s Province in 1874.
  2. Assam was declared as a Governor’s Province in 1921.

Select the correct statement/ statements using the codes given below. 

  1. I only
  2. II only
  3. Both I and II
  4. Neither I nor II

( Our APSC Prelim Test Series 2023 – Test 4-> Q59  &  Test 12 ->Q7 )

C. Both I and II

 

Q85. The Interim Budget -2024 announcement for implementation of 3 Economic Railway Corridors identified under the PM Gati Shakti for enabling multinodal connectivity includes which of the following?

  1. Energy, Mineral  and Cement Corridors
  2. Rural Roads Connectivity Corridors
  3. Port Connectivity Corridors
  4. High Traffic Density Corridors

Select the correct answer using the codes given below. 

  1. I and II only
  2. I, II and III only
  3. I, III and IV only
  4. II, III and IV only

C. I, III and IV only

 

Q86. Which of the following best describes/describe the main aim of ‘ Green India Mission’  of the government of India? 

  1. Incorporating environmental benefits and costs into the Union and state’s budgets thereby implementing the ‘ green accounting’ 
  2. Launching the second Green Revolution to enhance agricultural output so as to ensure food security to all and one in the future. 
  3. Restoring and enhancing forest cover responding to climate change by a combination of adaptation and mitigation measures. 

Select the correct answer using the codes given below. 

  1. I only
  2. II and III only
  3. III only
  4. I, II and III

C. III only

 

Q87. What is ‘Greenhouse Gas Protocol’? 

  1. It is an international accounting tool for government and business leaders to understand, quantify, and manage greenhouse gas emission.
  2. It is an initiative of the United Nations to offer financial incentives to developing countries to reduce greenhouse gas emissions and to adopt eco-friendly technologies. 
  3. It is an intergovernmental agreement ratified by all the member countries of the United Nations to reduce greenhouse gas emissions to specified levels by the year 2030. 
  4. It is one of the multilateral REDD+ initiatives hosted by the World Bank. 


A. It is an international accounting tool for government and business leaders to understand, quantify, and manage greenhouse gas emission.

 

Q88. Which of the following Faunas are found in North East India?

  1. Tokay gecko
  2. Red Panda
  3. Hoolock gibbon
  4. Lion-tailed macaque

Select the correct answer using the codes given below. 

  1. I and II
  2. II and III
  3. I, II and III
  4. I, II, III and IV


C. I, II and III

 

Q89. Which of the following national parks in India are World Heritage Sites? 

  1. Keoladeo National Park 
  2. Manas National Park 
  3. Nanda Devi Biosphere Reserve
  4. Great Himalayan National Park

Select the correct answer using the codes given below. 

  1. I, II and III only 
  2. II, III and IV only
  3. I, III and IV only
  4. I, II, III and IV


D. I, II, III and IV

 

Q90. Protection of Wildlife comes under which of the following in the Constitution of India?

  1. Fundamental Duties
  2. Directive Principles
  3. Seventh Schedule
  4. Fundamental Rights

Select the correct answer using the codes given below. 

  1. I and II
  2. II and IV
  3. I and III
  4. I, II and III


D. I, II and III

 

Q91. What is the name of the feature that allows organisms to survive in the conditions of their habitat?

  1. Adjustment
  2. Adaptation
  3. Acclimatization 
  4. Adaptive Variation 


B. Adaptation

 

Q92. Recently the ‘ Annual Status of Education Report (ASER), 2023 was submitted by PRATHAM. Which of the following statements is/are correct about the report?

  1. The Report focused on 14-18 years of age group. 
  2. According to the report, the percentage of the youths not enrolled in any form of education has gone up for 14 years olds and gone down 18 years old as compared to the year 2017. 
  3. For discontinuing education, almost 20% of girls attributed it to family constraints’. 
  4. The report asserted that the fear that the several other students dropped out of school during the COVID-19 Pandemic due to loss of livelihood was true. 

Select the correct answer using the codes given below.

  1. I only
  2. I and II
  3. I and III
  4. II and IV

C. I and III

Q93. Consider the following statements regarding Bhimbor Deuri, a prominent leader of the Tribal League:

  1. Bhimbor Deuri vehemently opposed the Muslims League’s conspiracy to include Assam in a grouping plan with Bengal.
  2. Bhimbor Deuri supported a proposal for a separate electorate  for the tribal people for strengthening the rights of indigenous people. 

Select the correct statement/ statements using the codes given below. 

  1. I only
  2. II only
  3. Both I and II
  4. Neither I nor II

C. Both I and II


Q94. Match the following folk dance forms (List-I) with their corresponding communities of Assam (List-II):

List-I                                         List-II             

  1. Hacha Kekan              1. Dimasa
  2. Hanaghora                  2. Karbi 
  3. Bisuyo Jama Dance  3. Rabha
  4. Baidima                       4. Deuri


D. a2, d3, c4, d1

 

Q95. Which of the following pairs is /are incorrectly matched?

  1. Finance Commission of India: Constitutional body
  2. Securities and Exchange Board of India: Statutory body
  3. NITI Aayog: Constitutional body
  4. Reserve Bank of India: Statutory body

Select the correct answer using the codes given below.

  1. I and III only
  2. II and IV only
  3. III only
  4. IV only

C. III only

 

Q96. Which of the following groups of places is/are located on the North bank of Brahmaputra?

  1. Bijni Jogighopa, Raha, Titabor
  2. Hajo, Sarthebari, Moran Bihpuria
  3. Bongaigaon, Dekhla-juli, Gohpur
  4. Sualkuchi, Barpeta, Kharupetia, Narayanpur

Select the correct answer using the codes given below.

  1. I and II
  2. III and IV
  3. I and III
  4. IV only


B. III and IV only

Q97. Match the following personalities (List-I) with the popular sobriquets by which they are known (List-II):

List- I                                               List-II

  1. Anandiram Das                 1. Geetikavi
  2. Parbati Prasad Baruah    2. Lauha Manab
  3. Bishnuram Medhi            3. Bonkonwar
  4. Hemchandra Baruah      4. Tyagbir

Select the correct answer using the codes given below.

D. a3, b1, c2, d4

 

Q98. Which of the following is/are correct about ‘eRupee’?

  1. It refers to Rupee circulation in Europe. 
  2. It refers to Exchange rate of Rupee. 
  3. It is digital Rupee App of the State Bank of India. 
  4. It is the Central Bank Digital Currency. 

Select the correct answer using the codes given below.

  1. I and II
  2. I and IV
  3. III only
  4. IV only

C. III only

Q99. The proposed India-Middle East-Europe Economic Corridor (IMEC) is expected to have multiple benefits for India. Which of the following is/are in the lists of benefits?

  1. It is a new and more efficient way for India to export its goods to Europe. 
  2. It would make India a more attractive destination for foreign investment.
  3. The construction and maintenance of the Economic corridor would create millions of new jobs in India. 
  4. The construction of the new corridor will be under the supervision of the World Trade Organisations. 

Select the correct answer using the codes given below.

  1. I only
  2. I and II
  3. I, II and III
  4. I, II and IV only


C. I, II and III

 

Q100. Match the following places (List-I) with their districts where these are located (List-II):

List-I                              List-II

  1. Sitajakhala          1. Dima Hasao
  2. Silghat                 2. Nagaon
  3. Kulsi                     3. Kamrup 
  4. Semkhor            4. Morigaon

Select the correct answer using the codes given below.

C. a4, b3, c2, d1

Go to APSC Prelim Previous Years Questions

Assam 2022 Yearbook – Important Current Affairs & GK (Important for APSC & other exams)

Assam 2022 Yearbook – Important Current Affairs & GK (Important for APSC & other exams)

Current Affairs 2022 Yearbook PDF

Current Affairs 2022 Yearbook – Important Current Affairs & GK (Important for APSC & other exams)

Current Affairs 2022 Yearbook comprehensively covers all the important events/topics of Current affairs of Assam, North-East, India and the World, important for competitive exams point of view.

Total 247 pages.

Click to get Current Affairs 2022 Yearbook PDF

Go to Assam Current Affairs

Assam 2023 Yearbook – Important Current Affairs & GK (Important for APSC & other exams)

Assam 2023 Yearbook – Important Current Affairs & GK (Important for APSC & other exams)

Current Affairs 2023 Yearbook PDF

Current Affairs 2023 Yearbook – Important Current Affairs & GK (Important for APSC & other exams)

Current Affairs 2023 Yearbook comprehensively covers all the important events/topics of Current affairs of Assam, North-East, India and the World, important for competitive exams point of view.

Total 181 pages.

Click to get Current Affairs 2023 Yearbook PDF

Go to Assam Current Affairs

Assam Current Affairs & GK Quiz for APSC 2024 – Set 351: January 2024, Week 3

Assam Current Affairs & GK Quiz for APSC 2024 – Set 351: January 2024, Week 3

(Daily and Monthly Assam Current Affairs & GK Quiz & MCQ for APSC Exams and various Assam exams)

Assam Current Affairs Quiz Home Page

Current Affairs Assam Quiz - Assam exam

Q1. Which among the following Tiger Reserve has got the title of India’s first ‘Dark Sky Park’?

  1. Pench Tiger Reserve
  2. Manas Tiger Reserve
  3. Pakke Tiger Reserve
  4. Nagarjunsagar-Srisailam Tiger Reserve

Pench Tiger Reserve

The Pench Tiger Reserve (PTR) in Maharashtra has been marked as India’s first Dark Sky Park and the fifth Dark Sky Park in Asia for protecting the night sky and preventing light pollution.

The reserve’s certification recognizes its commitment to protecting the night sky, reducing light pollution, and creating an ideal environment for astronomy enthusiasts and stargazers.

 

Q2. Which bank signed a MoU with the Ministry of Rural Development (MoRD) to provide loans to rural Self-Help Groups (SHGs)?

  1. Canara Bank
  2. Punjab National Bank
  3. State Bank of India
  4. Union Bank of India

State Bank of India

State Bank of India introduced a specialized financial product, “Svyam Siddha”, exclusively tailored for SHG women entrepreneurs seeking loans up to Rs. 5 lakhs. 

This initiative is uniquely designed to alleviate the challenges associated with extensive documentation requirements for bank loan applications and reduce the Turn Around Time (TAT).

 

Q3. Which state government has launched the MukhyaMantri Mahila Udyamita Abhiyaan aimed at empowering rural women entrepreneurs?

  1. Gujarat
  2. Uttar Pradesh
  3. Bihar
  4. Assam

Assam

The Assam government launched the Mukhyamantri Mahila Udyamita Abhiyaan (MMUA) to empower rural women entrepreneurs. 

This financial assistance scheme targets women beyond Self-Help Groups, emphasizing family planning for eligibility. The goal is to transform participants into “Rural Micro Entrepreneurs” with a ₹1 lakh annual income target.

 

Q4. India and which country has partnered to launch Green Fuels Alliance India (GFAI)?

  1. USA
  2. Denmark
  3. UAE
  4. Germany

Denmark

  • Green Fuels Alliance India (GFAI) is an alliance between Denmark and India to boost collaborative efforts in the sustainable energy solutions sector.
  • It aims to advance the green fuels sector, including green hydrogen, through innovation and partnerships.
  • The new alliance is a strategic initiative led by the Danish Embassy and Consulate General of Denmark in India.

 

Q5. ‘Sinomicrurus Gorei’ a new species of coral snake was discovered in

  1. Arunachal Pradesh
  2. Mizoram
  3. Madhya Pradesh
  4. West Bengal

Mizoram

Researchers from Mizoram University’s zoology department have discovered a new species of coral snake in the state.  It has been named Sinomicrurus Gorei after British India doctor Gore.

The reptile is locally known as ‘Rulṭhihna’ due to its striking resemblance to a Mizo traditional amber necklace called ‘Ṭhihna’.

 

Q6. “Panchayats Free From Gender-Based Violence – A Handbook for Elected Representatives” is released by

  1. Narendra Modi
  2. Amit Shah Shri
  3. Giriraj Singh
  4. Dharmendra Pradhan

Giriraj Singh

 

Q7. Who among the following has become France’s youngest prime minister

  1. Gabriel Attal
  2. Hassanal Bolkiah
  3. Robert Fico
  4. Srettha Thavisin

Gabriel Attal

 

Q8. The Union Minister of Ayush and Ports, Shipping & Waterways Shri Sarbananda Sonowal laid the foundation stone for ‘Ayush Diksha’, centre in 

  1. Guwahati
  2. Prayagraj
  3. Bhubaneswar
  4. Agra

Bhubaneswar

  • Shri Sarbananda Sonowal inaugurated ‘Ayush Diksha,’ a groundbreaking center at the Central Ayurveda Research Institute in Bhubaneswar.
  • The ‘Ayush Diksha’ center is set to become a hub for the advancement of human resources in Ayush, with a primary focus on Ayurveda professionals. 
  • The institution aims to foster collaboration with prominent national institutes to enhance capacity, strengthen human resources, facilitate research and development, and achieve self-sustainability for revenue generation.

 

Q9. Lanjia Saura paintings, which recently got GI tag, belong to which state?

  1. Assam
  2. Gujarat
  3. Maharashtra
  4. Odisha

Odisha

Lanjia Saura paintings, also known as ekons or iditals, are a unique style of wall mural art from Odisha. The art form belongs to the Lanjia Saura community, a PVTG largely residing in the Rayagada district. These paintings are in the form of exterior murals painted on the mud walls of homes.

Q10. Who has become the second Indian to win a gold medal in the Asian Marathon Championship?

  1. Balraj Kaushik
  2. Gaurav Karjee
  3. Man Singh
  4. Dheeraj Singha

Man Singh

  • Man Singh has become only the second Indian male athlete to win a gold medal in the Asian Marathon Championships. 
  • He won the gold medal by timing 2 hours, 14 minutes and 19 seconds. 
  • Gopi Thonakal was the first Indian male athlete to win the Asian Marathon Championship title in the year 2017.

 

Q11. Who among the following is the Chief architect of the Ram Temple?

  1. Chandrakant Bhai Sompura
  2. Ashok Singhal
  3. Arun Yogiraj
  4. Ashok Bhushan

Chandrakant Bhai Sompura

Chandrakant Sompura, the chief architect of the Ayodhya Ram Mandir, hails from a distinguished lineage of temple architects based in Ahmedabad.

Notable among their creations are iconic structures like the Somnath Temple in Gujarat, the Swaminarayan temple in Mumbai, the Akshardham temple complex in Gujarat, and the Birla Temple in Kolkata.

 

Q12. Ministry of Electronics and Information Technology (MeitY) inaugurated the India Innovation Centre for Graphene (IICG) and a Centre of Excellence (CoE) in Intelligent Internet of Things (IIoT) Sensors in

  1. Maharashtra
  2. Kerala
  3. Arunachal Pradesh
  4. Rajasthan

Kerala

Ministry of Electronics and Information Technology (MeitY), launched two flagship programs – “Centre of Excellence (CoE) in Intelligent Internet of Things (IIoT) Sensors” and India’s first Graphene Centre “India Innovation Centre for Graphene (IICG)” today at Maker Village, Kochi, Kerala.

 

Q13. Which airport was the venue for the Wings India 2024 event?

  1. Chhatrapati Shivaji Maharaj International Airport
  2. Lokpriya Gopinath Bordoloi International Airport
  3. Sardar Vallabhbhai Patel International Airport
  4. Begumpet Airport, Hyderabad

Begumpet Airport, Hyderabad

  • Union Minister for Civil Aviation Shri Jyotiraditya M Scindia inaugurated Wings India 2024, Asia’s largest aviation expo, at Begumpet Airport in Hyderabad.
  • The Airports Authority of India, in collaboration with the Ministry of Civil Aviation and FICCI, is organizing the biennial (after every two years) event. 
  • Wings India 2024 showcases a collection of new-generation aircraft displays, allied aviation services, auxiliary unit industries, and advancements in the tourism sector. 
  • The theme of Wings India 2024 is “Connecting India to the World in Amrit Kaal: Setting the Stage for India Civil Aviation @2047”.

 

Q14. Which state government has the Yuva Nidhi Scheme for Unemployed Youth?

  1. Uttar Pradesh
  2. Assam
  3. Karnataka
  4. West Bengal

Karnataka

  • Chief Minister Siddaramaiah inaugurated Yuva Nidhi, a scheme providing financial aid to unemployed graduates and diploma holders for two years.
  • Under the scheme, the government has committed to providing Rs 3,000 per month to degree holders and Rs 1,500 per month to diploma holders who have been unable to secure employment in the last six months and are not pursuing higher studies.

 

Q15. Who has been selected for Assam’s highest civilian award ‘Assam Vaibhav’ for 2023?

  1. Ranjan Gogoi
  2. Pranabjyoti Deka
  3. Pranjal Saikia
  4. Ratan Tata

Ranjan Gogoi

Former Chief Justice of India and current Rajya Sabha MP, Ranjan Gogoi, is set to be honored with the prestigious ‘Assam Baibhav’ award, the highest civilian award in Assam.

Ranjan Gogoi, the 46th Chief Justice of India and a Rajya Sabha MP, is the first from Northeast India to hold the CJI post. 

The Chief Minister also credited Ranjan Gogoi for his role in the establishment of the Ram Temple in Ayodhya, emphasizing the significant impact of Gogoi’s Assamese roots on this historic development.

Go to Assam Current Affairs Quiz Home Page